Test E

Ace your homework & exams now with Quizwiz!

1.) A woman witnesses a robbery in the park. What is she most likely to remember about this incident a week later? a. She will recall the events that took place the day of the robbery. b. She will remember the event in terms of other incidents of robbery witnessed before. c. She will remember the events that took place right after the robbery. d. She will recall the events that took place immediately before and during the robbery.

a. She will recall the events that took place immediately before and during the robbery. b. This question concerns a "flashbulb memory," a distinctive type of episodic memory that is related to a significant or traumatic event. Such memories are characterized by vivid detail of the event, and are likely to be maintained over long periods of time.

1.) To help a child overcome a phobia, you would most likely: a. Follow live or symbolic modeling with guided participation b. Follow live modeling with symbolic modeling c. Follow participant modeling with live modeling d. Use guided participation before live or symbolic modeling

Follow live or symbolic modeling with guided participation a. When using modeling the typical sequence is that symbolic modeling (e.g., watching a filmed model) or live modeling (e.g., watching a live model) is followed by participant modeling. Participant modeling involves live modeling plus contact with the therapist (or model) who gradually guides the person in activities involving physically interacting with the feared object.

1.) The benefits of flextime are thought to come from employees: a. experiencing more freedom and control. b. being able to work fewer total hours each week. c. enjoying the benefits of a shorter work week and longer weekend. d. feeling more responsibility and challenge on their jobs.

a. experiencing more freedom and control. a. The benefits of flextime seem to be related to an increased sense of personal freedom and control. When flextime is implemented, the number of hours worked each week remains the same (Response 2) although there is flexibility with regard to the specific hours that people work. Response 3 refers to the compressed work-week. Response 4 describes the benefits of job enrichment.

1.) ADHD affects about: a. 1 out of 5 children b. 1 out of 10 children c. 1 out of 20 children d. 1 out of 50 children

a. 1 out of 20 children a. According to the DSM-5, Attention-Deficit/Hyperactivity Disorder (ADHD) is estimated to affect about 5% of children, or about 1 in 20 children (Response 3). If you were unfamiliar with the prevalence of ADHD you might have been able to eliminate Response 1 which is too high (1 out of 5 is 20% of all children), and Response 4 which is too low (1 out of 50 is only 2% of all children). The prevalence rate of ADHD in adults is 2.5%. The DSM-5 also notes that ADHD occurs more commonly in males than females, with a ratio of about 2:1.

1.) To qualify for a diagnosis of mental retardation, a person must show significant deficits in adaptive functioning (e.g., communication, self-care) as well as subaverage intellectual functioning, with an onset prior to age 18. Subaverage intellectual functioning is defined as an IQ score of: a. Below 85 b. 70 or below c. 50 or below d. Below 25

a. 70 or below a. According to the DSM-5, one of the criteria for the diagnosis of intellectual disability is an IQ score approximately two standard deviations or more below the population mean. Both the WISC and the Stanford-Binet have a mean of 100 and a standard deviation of 15. Thus, a score of 70 is equivalent to two standard deviations below the mean

1.) Your score on the EPPP is two standard deviations above the mean. You have scored at the: a. 84th percentile rank b. 90th percentile rank c. 95th percentile rank d. 98th percentile rank

a. 98th percentile rank b. Congratulations! An EPPP score that is two standard deviations above the mean is exceptional. If you scored at the mean you would be at the 50th percentile rank. If you scored one standard deviation above the mean, you would be at the 84th percentile rank (Response 1). At two standard deviations above the mean, your score is at the 97.5th or roughly the 98th percentile rank. You may have recognized 95% (similar to Response 3) as the confidence interval that corresponds to two standard deviations above and below the mean. This question, however, was asking about percentile ranks not confidence intervals

1.) Your client is an Aries and describes herself as adventurous, confident, generous, and calm. While astrological descriptions note that Aries are in fact adventurous and confident, Aries are also described as selfish and quick-tempered, which this client ignores. The fact that this client strongly believes in the accuracy of her astrological description is an example of: a. A confirmation bias b. A selection bias c. A self-fulfilling prophecy d. A congruence bias

a. A confirmation bias a. A confirmation bias is the tendency to only look at confirmatory evidence while ignoring contradictory evidence. In this scenario, your client attends to and agrees with statements about Aries being adventurous and confident, while ignoring statements about Aries being selfish and quick-tempered. A congruence bias (Response 4) is present when a person only directly tests a hypothesis and does not indirectly test the hypothesis. For example, to fully test the hypothesis that Aries is the accurate astrological description, the person would need to see both how closely she resembles Aries (directly test the hypothesis) as well as see that she does not resemble any other astrological sign (indirectly test the hypothesis). Selection bias (Response 2) refers to a threat to internal validity that occurs when subjects are not randomly assigned to treatment groups. A self-fulfilling prophecy (Response 3) refers to the process whereby people's expectations influence their behavior and the resulting outcomes

1.) A prescription for psychotropic medication is most likely to be written by: a. A psychiatrist in an outpatient clinic b. A psychiatrist in an inpatient hospital c. A psychiatrist in private practice d. A general practitioner

a. A general practitioner b. A recent study conducted by the federal Substance Abuse and Mental Health Services Administration (SAMSA) examined the prescribing patters for psychotropic medications (e.g., antidepressants, stimulants, etc) from August 2006 through July 2007. In that one year, of the 472 million prescriptions written for psychotropic medications 59% were written by general practitioners (e.g., internists, pediatricians, etc.).

1.) What does the term "minority marginalization" describe? a. A person who only associates with other persons from his or her culture, and is marginalized from the majority culture. b. A person who in marginalized from both his or her culture and the majority culture, yet associates with persons from other cultural groups. c. A person who only associates with persons from the majority culture, and is marginalized from his or her culture. d. A person who neither associates with persons from his or her own culture nor with persons from the majority culture.

a. A person who neither associates with persons from his or her own culture nor with persons from the majority culture. b. In marginalization, a person is alienated both from his or her own culture and the dominant culture.

1.) A gay man who is in the stage of identity pride is most likely to identify with which of the following? a. A sense of his homosexuality as better than another's heterosexuality. b. A tolerance of his homosexuality, while seeing heterosexuality as preferred. c. A feeling that neither homosexuality not heterosexuality is better, but rather both are valued. d. A feeling that heterosexuality is definitely preferred, and homosexuality is devalued.

a. A sense of his homosexuality as better than another's heterosexuality. a. As described in one model of sexual orientation identity development, people progress through six stages. The first stage is identity confusion. The second stage, identity comparison, is characterized by feelings of alienation. In the third stage, identity tolerance, homosexuality is tolerated. During the fourth stage, identity acceptance, homosexuality is seen as valid. In the fifth stage, identity pride, heterosexuality is devalued and homosexuality is preferred. The final stage is identity synthesis, in which sexual orientation is integrated. Response 2 is characteristic of identity tolerance. Response 3 could occur at the stage of identity acceptance, or during the final stage of identity synthesis. Response 4 is most likely to occur in the first two stages of identity confusion and identity comparison. Hint: Stages with the word "pride" (whether racial or sexual) usually involve feeling good about oneself and devaluing others.

1.) A man presents to a therapist's office, complaining that he has trouble making up his mind about his career and hobbies. He explains that he often procrastinates, can't stick with one thing, loses focus, and that his interests shift frequently. In addition, he experiences feelings of inadequacy. These are most likely symptoms of: a. ADHD. b. Schizoid Personality Disorder. c. Borderline Personality Disorder. d. Major Depressive Disorder, Recurrent.

a. ADHD. a. The symptoms described most closely resemble the symptoms of ADHD. The man described in the question has difficulty focusing, procrastinates, and is essentially unable to finish tasks, which make up some of the residual symptoms of ADHD that persist into adulthood. Schizoid Personality Disorder (Response 2) is primarily characterized by a lack of interest in relationships, detachment, and a restricted range of emotions. Borderline Personality Disorder (Response 3) involves instability in affect, relationships and self-image. While Major Depressive Disorder, Recurrent (Response 4) can include indecisiveness and feelings of worthlessness, the clinical picture is typically much more serious with reports of depressed mood, loss of interest or pleasure, appetite and sleep disturbance, etc.

1.) A 32-year-old African American female psychologist who works in a community mental health center conducts an intake on a 51-year-old married Native American male. Although she has availability in her schedule to treat him, she has no prior clinical experience working with Native Americans. As an ethical psychologist she would be best advised to: a. Refer the patient to a spiritual healer b. Discuss with the patient how he feels and any concerns he may have about working with a non-Native American therapist c. Acquire appropriate consultation and/or supervision in order to treat him d. Refer the patient for treatment to a Native American therapist at the community mental health center, if available

a. Acquire appropriate consultation and/or supervision in order to treat him The APA Ethics Code notes that when factors associated with ethnicity, culture, or national origin may be important to treatment, psychologists either have or obtain training, experience, consultation or supervision necessary to ensure competence, or they make appropriate referrals. Essentially, the Ethics Code encourages psychologists, whenever possible, to acquire competence in treating diverse populations. Ethnic-minorities clients are not required to be seen only by therapists of. The same ethnicity (ruling out Response 4). Nor at this point would it be appropriate to refer the patient to a spiritual healer (ruling out Response 1). While it would be important to discuss the patient's feelings with him and any concerns he may have (Response 2) this, in and of itself, does not sufficiently address the ethical issues involved related to acquiring competence

1.) All of the following are commonly treated using principles of reciprocal inhibition except: a. Specific phobias b. Smoking c. Paraphilias d. Agoraphobia

a. Agoraphobia b. The treatment of choice for agoraphobia is flooding, which is based on the principle of extinction; the person confronts the feared situation (CS) until it no longer triggers an anxiety response. Reciprocal inhibition underlies counterconditioning, which states that two incompatible responses (e.g., fear and relaxation) cannot occur at the same time. Specific phobias (Response 1) are commonly treated with systematic desensitization, a form of counterconditioning, although treatments for specific phobias that involve direct exposure (e.g., flooding) are superior. Smoking (Response 2) and paraphilias (Response 3) are treated with aversive conditioning, also a form of counterconditioning.

1.) A patient experiences seizures in an unknown area of his brain. What diagnostic test would his doctor be most likely to conduct during a seizure? a. An MRI b. A PET scan c. An EEG recording d. CT scan

a. An EEG recording a. The best way to localize the source of a seizure by doing an EEG recording during the seizure (Response 3). MRI (magnetic resonance imaging) scans (Response 1) use radio waves to determine brain structure and degree of pathology. The MRI is a superior tool for creating images of soft tissue such as brain matter. The CT scan (Response 4) provides a detailed x-ray of bone structure. PET (positron emission tomography) scans (Response 2) provide information on brain functioning using dyes that link up with brain chemicals such as glucose

1.) A person who has been diagnosed with terminal cancer asks, "Why me?" According to Kubler-Ross, this is characteristic of which emotional stage? a. Denial b. Depression c. Bargaining d. Anger

a. Anger b. Kubler-Ross has proposed that most people go through five stages when they confront their own death: denial, anger, bargaining, depression, and acceptance (forming the acronym DABDA). "Why me?" is characteristic of the stage of anger, in which the person is questioning why he or she has to be the one to die. Denial (Response 1) is characterized by disbelief (e.g., "Not me" or "It's not possible"). In the depression phase (Response 2), the person experiences despair and hopelessness. In the bargaining phase (Response 3), the person tries to make deals in order to not have to go through the inevitable (e.g., "If I can beat this, I'll be nicer to my family and friends").

1.) You are conducting an experiment with a 2-year-old girl named Betsy. You should Betsy two dolls named Ann and David who are playing with a ball. When the dolls are done playing, they put the ball in a blue box. Then Ann leaves the room. While Ann is away, David plays with the ball again and puts it away in a red box. If you ask Betsy where Ann will look to find the ball when she returns to the room, what will Betsy say? a. Ann will look for the ball in the blue box b. Ann will look for the ball in the red box c. Ann will look for the ball in both boxes d. Ann won't look in either box for the ball

a. Ann will look for the ball in the red box a. This question is asking about a common experiment used to test for the theory of mind called the false-belief task. In this experiment you are trying to determine if Betsy has developed a sense of the independence of others' minds to the point that she can imagine that people can have other (false) beliefs about the world than she has. In order to correctly guess that Ann will look for the ball in the blue box (where she last left the ball), Betsy would have to have an understanding of how beliefs are formed and that people's beliefs can differ from reality (the reality that the ball is now in the red box). Research has found that the ability to correctly answer the false-belief experiment normally develops around the age of four. Therefore, two-year-old Betsy would answer that Ann would look in the red box (Response 2) because Betsy does not yet grasp the fact that just because she knows where the ball is, Ann does not

1.) A patient reports experiencing recurrent, intrusive thoughts about contamination. Consequently, she frequently engages in repetitive behaviors that seem to reduce her distress. She is completely convinced her thoughts and behaviors are reasonable, in spite of all evidence to the contrary. The class of medication that would be used to treat her are the: a. Mood stabilizers b. Antidepressants c. Anxiolytics d. Antipsychotics

a. Antidepressants a. These symptoms meet the criteria for Obsessive Compulsive Disorder (OCD). The DSM-5 has added the specifier "with absent insight/delusional beliefs" for individuals who are completely convinced that obsessive-compulsive disorder beliefs are true. Although OCD is an anxiety disorder, it is most commonly treated with antidepressants including Clomipramine (Anafranil), which is a tricyclic, or Prozac (fluoxetine), which is an SSRI (Response 2, correct answer).

1.) Which of the following best defines the model of antisocial behavior developed by G. R. Patterson? a. Antisocial behavior results from a predictable developmental sequence of experiences. b. Antisocial behavior is modeled by the parent(s) through excessive discipline. c. Antisocial behavior is genetically determined. d. Children learn antisocial behavior primarily through affiliation with deviant peer groups.

a. Antisocial behavior results from a predictable developmental sequence of experiences. a. G. R. Patterson has proposed that delinquency results from a reliable developmental sequence of experiences. First, the pattern begins with poor parenting, for example, threats, criticism, yelling, noncontingent punishment, noncontingent reinforcement, and reinforcement of the child's coercive behaviors within the family. This inept parenting can be due to the personality of the parent(s), the child's temperament, as well as a number of disruptors (e.g., disadvantaged SES, or stressors such as unemployment, marital discord or divorce). Over time, the intensity of coercive behaviors within the family escalates (e.g., hitting, physical attacks), and the child learns to control other family members through coercive means. Second, the child with conduct problems experiences academic failure and peer rejection. Third, as a result of feeling rejected, the child or adolescent becomes depressed and is more likely to get involved in a deviant peer group. According to Patterson, these three steps contribute to the development of chronic delinquent tendencies and behaviors. Response 2 is incorrect in that even though excessive parental discipline may contribute to antisocial behavior, it is not just through modeling, as this response suggests. Based on Patterson's model, aggression is learned, and is not innate (Response 3). Affiliation with deviant peer groups contributes to the development of aggression, however, it is one of the factors responsible, and not the only one (Response 4).

1.) A verbal test of intelligence would be appropriate as an estimate of overall intelligence for a child who: a. Attends a suburban elementary school b. Attends an inner city school c. Has a learning disorder d. Has a history of delinquency

a. Attends a suburban elementary school a. Intelligence test scores are affected by sociocultural and psychological factors. For example, people who are more educated and of higher socio-economic status tend to score higher on verbal tests of intelligence than those of lower SES and education. Therefore, a verbal test of intelligence would be a more accurate measurement of overall intelligence for a child from a suburban elementary school (Response 1) because people who lie in the suburbs generally are of higher SES and have better access to educational opportunities. Conversely, a child attending an inner city school (Response 2) would not perform optimally on a verbal test of intelligence; such a test would underestimate intelligence, and a performance-based test of intelligence would probably be a better measure of overall IQ. When a child has a learning disorder (Response 3), verbal test scores are frequently adversely affected. For example, a child with a reading disorder would probably have less well developed skills on subtests such as Vocabulary or information on the WISC-IV. Children with delinquency (Response 4) also tend to have had less exposure to educational opportunities; delinquency is therefore associated with lower scores on verbal tests of intelligence in comparison with performance-based tests of intelligence

1.) A 8-year-old boy is brought to see you for treatment by his parents. You are told that he has virtually no friends, has a hard time interacting with others, and does not like to try new activities. You notice, however, that he speaks without any difficulty, and shows no signs of cognitive deficits. You would most likely diagnose him with: a. Autism spectrum disorder b. Social (pragmatic communication disorder c. ADHD d. Oppositional defiant disorder

a. Autism spectrum disorder a. In the DSM-5 autism spectrum disorder is a new disorder that encompasses autistic disorder, Asperger's disorder, and childhood disintegrative disorder. Autism spectrum disorder is characterized by: a) persistent deficits in social communication and social interaction across multiple contexts; and b) restricted, repetitive patterns of behavior, interests, or activities (Response 1, correct answer). Language impairment and/or intellectual impairment are not required for the diagnosis. Social (pragmatic) communication disorder is also a new DSM-5 diagnosis. This disorder involves difficulties in the social use of verbal and nonverbal communication as manifest by all of the following: a) deficits in communication for social purposes (e.g., greeting), problems changing communication to match context (e.g., speaking differently in a classroom than on the playground); b) difficulties following rules of conversation (e.g., taking turns); and c) difficulty understanding nonliteral or ambiguous meanings (e.g., idioms, humor, metaphors). ADHD (Response 3) is characterized by a persistent pattern (at least 6 months) of inattention and/or hyperactivity-impulsivity. Oppositional defiant disorder (Response 4) involves a recurrent pattern of angry/irritable mood, argumentative/defiant behavior, or vindictiveness, which persists for at least 6 months, not described here

1.) The Taylor-Russell tables incorporate information about: a. Base rate, selection ratio, and criterion-related validity b. Base rate, selection ratio, and construct validity c. Base rate, selection ratio, incremental validity, and reliability d. Base rate, selection ratio, incremental validity, and external validity

a. Base rate, selection ratio, and criterion-related validity a. The Taylor-Russell tables is a book of tables that is used to determine the amount of improvement that occurs when a predictor test is used in making selection/hiring decisions as compared to when selection/hiring decisions are made without such a test. The amount of improvement is termed "incremental validity." The Taylor-Russell tables use data on the selection ratio, base rate, and criterion-related validity of the predictor test. The selection ratio is the proportion of openings to applicants. The base rate is the proportion of successful employees to total number of employees, before the introduction of the test. To optimize incremental validity, the selection ratio should be low (.10), and the base rate moderate (around .50). Construct validity (Response 2), reliability (Response 3), and external validity (Response 4) are all unrelated to the Taylor-Russell tables

1.) In a study by Rodin and Langer, a group of nursing home patients were given plants to care for. Compared to the control group, the treatment group showed: a. No differences in health and mortality rates b. Better health and lower mortality rates c. Better health and higher mortality rates d. Worse health and lower mortality rates

a. Better health and lower mortality rates a. Although you are unlikely to be familiar with this specific study, you might been able to arrive at the correct answer through a process of elimination. Mortality rate refers to rate of death. It is unlikely that better health would be paired with higher mortality rates, thus eliminating Response 3. It also seems unlikely that plants would worsen people's health, eliminating Response 4. If you happened to remember the research about how pets improve health, you might wager that plants would as well (Response 2). Of you might guess that a study on an obscure topic is unlikely to be included if no significant differences were found, thus eliminating Response 1

1.) A 16-year-old boy has recently begun to exhibit apathy, hypersexuality, poor concentration and little need for sleep. The most likely diagnosis for him would be: a. Borderline personality disorder b. Bipolar disorder c. Schizophrenia d. ADHD

a. Bipolar disorder a. These symptoms, in particular the reduced need for sleep and hypersexuality, suggest bipolar disorder. Given that the symptoms have started recently, a personality disorder (Response 1) can be ruled out. Schizophrenia (Response 3) is typically characterized by psychotic symptoms, such as delusions, hallucinations, bizarre speech and thoughts, none of which are mentioned here. ADHD (Response 4) can be ruled out for two reasons. First, the symptoms have a recent onset; for ADHD, they must have been present prior to age seven. Second, even though poor concentration can be a symptom of ADHD, hypersexuality and reduced need for sleep are not.

1.) Research on Cognitive Behavioral Therapy (CBT) with older adults has found that: a. CBT is generally less effective with older adults b. CBT is as effective with older adults as with other populations c. CBT is as effective with older adults but should be more focused and on briefer duration d. CBT is as effective with older adults but should progress more clearly and slowly

a. CBT is as effective with older adults but should progress more clearly and slowly b. Research indicates that in order for CBT to be effective with older adults, it must accommodate the major cognitive changes associated with aging. As a result, CBT with older adults should proceed at a slower pace with more frequent pauses and summaries

1.) A Caucasian teenager is knifed and killed near his home. Based on the research on adolescent homicide, his attacker most likely is: a. African American b. Hispanic American c. Caucasian d. Asian American

a. Caucasian a. Research has indicated that most homicides are intraracial, or occur between members of the same race. In other words, Caucasians are most likely to kill Caucasians African Americans are most likely to kill other African Americans, etc. In general, all types of crime (e.g., rape, burglary, etc.) tend to follow this intraracial pattern.

1.) Agranulocytosis is associated with which of the following medications? a. Clozapine. b. Thorazine. c. Phenelzine. d. Lithium Carbonate.

a. Clozapine. a. Agranulocytosis is a potentially lethal side effect associated with Clozapine (Clozaril), a novel antipsychotic. Agranulocytosis is a sudden drop in the granulocyte count, usually occurring within hours to 12 weeks of initial administration, and manifesting as a sore throat and high fever. Thorazine (Chlorpromazine) (Response 2) is a traditional antipsychotic and is not known to bring about agranulocytosis. Phenelzine (Nardil) (Response 3) is an MAOI. MAOIs should not be mixed with foods high in tyramine because they can induce a hypertensive emergency, which requires immediate medical attention. Symptoms of a hypertensive crisis include headache, stiff neck, sweating, nausea, and vomiting. The most severe side effect of Lithium Carbonate (Response 4) is lithium toxicity, which is potentially fatal and is always a medical emergency. Lithium toxicity may initially mimic the flu and can include vomiting, abdominal pain, severe diarrhea, severe tremor, ataxia, coma, seizures, confusion, and irregular heart beat.

1.) Research on the treatment of paraphilic disorder has found that: a. Cognitive behavioral therapy is most commonly used and is an evidence-based treatment b. Cognitive behavioral therapy is most commonly used but is not an evidence-based treatment c. Multisystemic therapy is most commonly used and is an evidence-based treatment d. Multisystemic therapy is most commonly used but is not an evidence-based treatment

a. Cognitive behavioral therapy is most commonly used but is not an evidence-based treatment a. Cognitive behavioral therapy (CBT) has been and continues to remain the predominant approach to the treatment of sex offenders and/or individuals with paraphilic disorder. CBT involves decreasing inappropriate sexual arousal (e.g., through aversive counterconditioning), enhancing appropriate sexual arousal to adult partners (e.g., through orgasmic reconditioning), and cognitive restructuring, assertive skills training, social skills training, enhancing empathy, and relapse prevention. Unfortunately, the evidence base for cognitive-behavioral treatment is extremely limited (Response 2, correct answer). Multisystemic therapy is an intensive family- and community-based treatment that focuses on home, school, work, and neighborhood to promote prosocial behavior while decreasing antisocial behavior. Multisystemic therapy has shown promise in the treatment of conduct disorder, but has not been widely used with individuals with paraphilic disorder

1.) You have been treating a patient in therapy for the past several months. This patient owes you money for several sessions, but has missed her appointments for the past six sessions. You receive a release of information signed by the patient requesting treatment information from another psychologist whom the patient recently started seeing. How should you respond to this situation? a. Refuse to send the information, but do not disclose the reason to the psychologist in order to protect the patient's confidentiality. b. Refuse to send records because the patient owes you for several sessions. c. Contact the patient and inform her that you will send the information after she pays you the amount she owes. d. Cooperate with the request and send the necessary information.

a. Cooperate with the request and send the necessary information. b. This is a tricky question because ethical and legal obligations differ. According to the 2002 APA Ethics Code, it is ethically permissible to withhold treatment records because of nonpayment of fees, except in emergency situations. However, the Health Insurance Portability and Accountability Act (HIPAA), which has the force of federal law, prohibits the withholding of treatment records due to nonpayment of fees. In this situation, you should comply with the law by releasing the records.

1.) Which of the following is the most effective method for reducing racial prejudice? a. Techniques that allow people to recognize and confront their own prejudices. b. Exposure to campaigns counteracting stereotypes. c. Cooperative contact with members of other races. d. Increased education about other races.

a. Cooperative contact with members of other races. a. Increased contact with members of other races is the most effective method of reducing racial prejudice. However, it is important that the nature of the contact be cooperative, and that the members of the different races have equal status in their interactions.

1.) The "best interests of the child" is the guiding principle in which of the following situations? a. Medication b. Custody c. placement in academic settings d. decision as to whether a child should testify in court

a. Custody a. The "best interests of the child" is the principle used in child custody cases. The "least restrictive environment" is the principle that guides placement of students in academic settings (Response 3). In other words, students should be placed in the least restrictive environment in which it is likely that their needs can be met (e.g., time with the resource specialist as opposed to placement in a special education classroom).

1.) Typically parents treat boys and girls: a. Differently from birth b. Differently from about age 6 months c. Differently from about age 1 year d. Similarly

a. Differently from birth a. The "Baby X" studies of the 1970s and 1980s showed that adults treat newborns differently based on their perception of the child's sex. In this research parents were introduced to infants who were "disguised" (e.g., a baby boy in a pink dress). Then the scientists observed how the adults treated the baby. When they thought the baby was a girl, they brought her a doll; with a supposed boy, they brought him a truck. Researchers have also found that from birth parents physically treating boys and girls differently; boys are handled more vigorously while girls are treated more gently. As children get older, parents reinforce gender roles by exhibiting gender-specific behaviors toward children and encouraging gender-specific activities in their children. For example, parents talk more to their young daughters, give them less autonomy and encourage them to help others, while encouraging boys from an early age to express certain types of emotions but not others (e.g., fearfulness)

1.) Two therapists have been co-facilitating a process group. During a group session, the two therapists experience a difference of opinion in response to an exchange between the group members. How should they handle their disagreement? a. Discuss it after the group session is over. b. Seek consultation to help settle the disagreement. c. Discuss it openly in the group. d. Ask the group how the disagreement should be handled.

a. Discuss it openly in the group. a. In group therapy, appropriate open conflict among the leaders is seen as positive and healthy, because it models appropriate problem-solving and conflict resolution for the members.

1.) Of the following, which is the primary symptom that allows one to differentiate between caffeine intoxication and anxiety? a. Tachycardia b. Diuresis c. Gastrointestinal distress d. Flushed face

a. Diuresis a. Caffeine intoxication and anxiety disorders such as panic disorder and generalized anxiety disorder share a number of common symptoms including tachycardia (a fast beating heart) (Response 10, gastrointestinal or stomach distress (Response 3), flushing of the face (Response 4), and feelings of restlessness, nervousness, trembling, or shaking. A significant symptomatic difference between caffeine intoxication and anxiety is that caffeine's diuretic properties cause increased urination (diuresis), while increased urination is not a symptom of anxiety (Response 2)

1.) Context dependent memory is illustrated by which of the following? a. A student is able to recall a written story in great detail because she focused intently on its meaning while reading it. b. A student who smoked marijuana while studying for an exam, does better on the exam when under the influence. c. During an exam, a student is better able to recall the information that he learned in the classroom than what he studied at home. d. A student who was feeling anxious while studying for the licensing exam performs better on the exam when she is anxious.

a. During an exam, a student is better able to recall the information that he learned in the classroom than what he studied at home. a. Studies have shown that it is easier to recall information in the same environment in which it was originally acquired. This is known as context dependent memory. Response 1 describes semantic memory. Response 2 describes state dependent memory, which is the tendency for people to remember material better if their state of mind matches that of when they learned it. Response 4 describes mood congruent memory, in which a similar mood acts as a retrieval cue for recall of past information.

1.) When is the impact of poor nutrition most detrimental? a. During infancy and early childhood b. During adolescence c. During old age d. During the perinatal period

a. During the perinatal period b. Malnutrition is most detrimental while the fetus is still in utero, at a time when significant physical growth and brain development is taking place. The perinatal period (Response 4) refers to the period from about three months before birth (when the fetus is about one pound) to one month after birth. While poor nutrition certainly is detrimental across the lifespan, it is most detrimental prior to birth.

1.) All of the following interventions are commonly used in cognitive-behavioral therapy approaches except: a. activity scheduling. b. behavioral rehearsal. c. modeling. d. Empathy

a. Empathy b. Empathy is not a specific intervention used by cognitive-behavioral therapists. It is a key characteristic of client-centered therapy, and an important aspect of Kohut's self-psychology. This is not to say that cognitive-behaviorists are unempathic! Cognitive behavioral therapists commonly use all of the other interventions. Activity scheduling, behavioral rehearsal, and modeling are predominantly behavioral in nature, and they can be used to test out new cognitions, or precipitate the development of new and more adaptive cognitions. Activity scheduling (Response 1) is an intervention used to mobilize clients who are withdrawn and inactive. Behavioral rehearsal (Response 2) involves practicing target behaviors. Modeling (Response 3) involves showing the client how to engage in the target behavior.

1.) Some have argued that the tragic explosion of the space shuttle Challenger in large part was due to the fact that the head team of officials in charge of making decisions for the Challenger blatantly disregarded the input from engineers. The engineers had alerted the team that joint seals in the rocket boosters might fail in cold weather. In retrospect, the best way to have averted this tragedy would have been to: a. Increase cohesion between the head team of officials and the engineers b. Add a strong leader to the head team of officials c. Encourage the head team of officials to entertain contradictory views d. Encourage the engineers to voice their concerns more strongly to the head team of officials

a. Encourage the head team of officials to entertain contradictory views a. The scenario presented describes groupthink. Groupthink occurs in highly cohesive groups (the team of officials), and involves the tendency for group members to think alike and lose their critical evaluative capacities. Here, the cohesive team of officials disregarded outside input from the engineers, input that was not consistent with their own views. Research suggests that the best way to combat groupthink is to have someone within the group take on the role of "devil's advocate." A devil's advocate takes on a contradictory position just for the sake of argument

1.) Your patient tells you that one of your colleagues has been having a sexual relationship with one of his patients. What would be the best course of action in this situation? a. Report the psychologist to an ethics board yourself as the code requires that you take action when you become aware of another psychologist's misconduct. b. Encourage your client to report the psychologist to an ethics board. c. Attempt to contact the psychologist first and confront him with the inappropriateness of his actions. d. Attempt to seek an informal resolution first, if possible, and if not, report your colleague to the board.

a. Encourage your client to report the psychologist to an ethics board. a. This is an example of a question with no really good response. Ideally you might discuss with your client options available to her. Perhaps she knows the colleague personally, in which case she may want to consider talking to him, or making a report. Even though the Ethics Code does stipulate that psychologists must take action when they become aware of ethical breaches by others (Response 1), the issue of patient-therapist confidentially takes precedence. This also rules out Responses 3 and 4, which would involve a breach of confidentiality. Additionally, these responses are weak because when a psychologist becomes aware of a colleague's sexual misconduct, neither confrontation nor an informal resolution is a viable option.

1.) According to Kohlberg's theory, a person in the conventional stage of morality would have difficulty understanding that rules are followed in order to: a. Avoid punishment b. Fulfill one's duty to society c. Enhance social welfare d. Gain approval

a. Enhance social welfare a. This response is characteristic of post-conventional morality, and would be difficult to grasp by someone who has not yet reached this stage. This stage is characterized by morality based on personal standards (Morality of Contract, Individual Rights and Democratically Accepted Laws), as well as universal principles of justice, equality and respect for life (Morality of Individual Principles of Conscience). At this level, a person would follow rules in order to enhance the well-being of others. Responses 2 and 4 are both characteristic of conventional morality; fulfilling one's duty is characteristic of the Law and Order Orientation, and gaining approval is characteristic of the Good Boy/Good Girl Orientation. Response 1 is characteristic of the preconventional stage (Punishment-Obedience Orientation), therefore, someone at the conventional stage of morality would likely understand the perspective involved in the previous stage (preconventional).

1.) Which of the following factors is least likely to contribute to a reduction of aggression? a. Reduction of frustration levels. b. Enhancement of deindividuation. c. Teaching alternative behavioral responses. d. Superordinate goals.

a. Enhancement of deindividuation. a. Deindividuation refers to the process of suspending one's private self-identity and adopting the identity of the group, with decreased self-awareness and self-regulation (e.g., being a part of a mob). Since this process involves decreased self-regulation and awareness, increasing deindividuation could actually increase aggression. Reduction of frustration (Response 1) and teaching alternatives to aggression (Response 3) would serve to reduce aggressive responses. Superordinate goals (Response 4) would help to reduce conflict and aggressive responses, since they require cooperation among rivals in order to achieve the goals.

1.) A prisoner of war is told he will be tortured unless he reveals military secrets. He refuses. After an hour of torture, he continues to refuse. After two more hours, he finally reveals the secrets. This scenario depicts: a. Escape behavior b. Avoidance behavior c. Positive punishment d. Negative punishment

a. Escape behavior a. This situation is describing an escape paradigm, in that the prisoner of war is tortured, and then reveals military secrets in order to get the torture to stop. Note that had he immediately complied, the same behavior on his part (revealing the secrets) would have been avoidance behavior (Response 2), in that he could have avoided the torture altogether. Positive punishment (Response 3) involves applying an aversive stimulus after a target behavior has taken place, with the idea of reducing the target behavior. In negative punishment (Response 4) something desirable is taken away after an unwanted target behavior has occurred, with the goal of reducing the target behavior.

1.) An inverted U relationship can be interpreted with which of the following coefficients? a. Eta. b. Pearson r. c. Biserial. d. Tetrachoric.

a. Eta. a. Eta is the coefficient of choice for a curvilinear relationship. The pearson r (Response 2) is used to assess the strength and direction of a linear relationship between two continuous (interval/ratio) variables. The biserial coefficient (Response 3) is appropriate for assessing the relationship between a continuous variable and an artificially dichotomous variable. The tetrachoric coefficient (Response 4) is used to look at the correlation between two dichotomous variables.

1.) During a woman's first therapy session, it becomes clear that she would probably benefit more from couples therapy. As you evaluate her further, you realize that her husband was your boyfriend 25 years ago. How should you respond? a. Let both the husband and wife know why you cannot see the couple in marital therapy b. Continue to treat the woman in individual therapy and refer out the couple for marital therapy c. Explain to the woman why you cannot see her and her husband and make a referral for couples therapy d. Explain to the woman that you cannot see her and her husband in couples therapy, but do not tell her why in order to protect confidentiality; then make a referral for couples therapy

a. Explain to the woman why you cannot see her and her husband and make a referral for couples therapy a. The best course of action here would be to refer the woman and her husband to another therapist. It would also be important to explain to this woman that you have a prior connection with her husband, and that therefore it would not be appropriate for you to treat them. Since only the wife has come in to see you, discussing anything with her husband at this point would be a breach of her patient-therapist confidentiality (ruling out Response 1). It would not be appropriate for you to treat this woman in individual therapy (Response 2) for two reasons. First, it is likely you would not be able to remain objective, and second, you have determined that she would most benefit from couples therapy. As confidentiality only applies to the therapist-patient relationship, confidentiality would not apply to your ex-boyfriend (ruling out Response 4)

1.) A problem with using extinction is reduce self-injurious behavior is that: a. Extinction tends not to work for self-harm behavior b. Extinction is a classical conditioning technique while most self injurious behavior is under the control of operant conditioning c. Extinction does not create an immediate reduction of behavior d. Extinction works for certain self-injurious behaviors (e.g., head banging) but not others

a. Extinction does not create an immediate reduction of behavior a. While extinction may work to reduce self-injurious behavior (ruling out Responses 1 and 4), it does not create an immediate reduction in behavior. In fact, the use of operant extinction is typically associated with an initial increase in target behavior (a response burst) which could potentially life-threatening in the case of self-injurious behavior such as head banging or cutting. Extinction is both a classical and operant conditioning technique (ruling out Response 2). In classical conditioning, extinction is brought about by presenting the conditioned stimulus (CS) without the unconditioned stimulus (US), while in operant conditioning, extinction is brought about by withholding reinforcement

1.) Which of the following is the best predictor of suicide? a. Preoccupation with death b. Giving away one's possessions c. Feeling extreme hopelessness d. A family history of suicide

a. Feeling extreme hopelessness a. The best predictor of suicide is extreme hopelessness (Response 3) that typically co-occurs with depression and persistent suicidal ideation. While hopeless, the person feels there are a few options left other than that. A family history of suicide (Response 4) is a clear risk factor for suicide, however, in of itself family history does not predict suicide. Most people who have a relative that committed suicide, do not commit suicide themselves: conversely most people who do commit suicide do not have a family history of suicide. Some people do give away their possessions (Response 2) prior to committing suicide, however, this is not the norm. Preoccupation with death (Response 1) could reflect A number of issues depending on the way one is thinking about death. For example, a person who is preoccupied with thoughts of a recent death of a loved one, or is preoccupied with death because of a terminal diagnosis, is clearly not suicidal.

1.) Better prognosis for schizophrenia has been associated with which of the following factors? a. Insidious onset b. Early onset c. Negative symptoms d. Female gender

a. Female gender b. Some of the factors associated with better prognosis for schizophrenia include late onset of the illness, paranoid type (Response 1), positive symptoms (Response 3), and female gender (Response 4).

1.) You are interested in conducting research on cultural values toward older adults in the rural South. Upon consultation with expert in this area you find that asking for written informed consent might alienate some of your research subjects. You should: a. Go ahead and get written consent anyway, as it is required by the APA Ethics Code b. Find another way of ensuring consent and proceed with the research c. Find a different population to conduct your research on d. Recruit research subjects who are willing to give written consent

a. Find another way of ensuring consent and proceed with the research a. According to Section 3.10 of the APA Ethics Code regarding informed consent, "psychologists [must] appropriately document written or oral consent, permission and assent." Thus, written consent is not required by the APA Ethics Code and you can find another way of ensuring consent (e.g., oral consent). In this situation, for example, some of your subjects may be illiterate and insisting on written consent might jeopardize your research

1.) A procedure that compares an employee to all other employees in a work group is called: a. BARS b. Graphic rating c. Forced distribution d. Forced choice

a. Forced distribution a. Forced distribution is a comparative method of employee evaluation in which people are ranked to fit a given distribution. The other methods listed are all absolute or individual methods of evaluation. Forced choice (Response 4) is a method of evaluation in which the rater must choose between two seemingly equally desirable choices. In BARS, or behaviorally anchored rating scales (Response 1), behavioral anchors based on critical incidents are used to evaluate a person's performance. Graphic rating scales (Response 2) are likert scales that involve rating each employee on a variety of job dimensions.

1.) According to Sandra Bem's Gender Schema Theory: a. Gender roles are predominantly innately determined b. Gender roles are predominantly determined by a combination of innate and environmental factors c. Gender roles are predominantly determined by social learning d. Gender roles tend to be static over time

a. Gender roles are predominantly determined by social learning a. Sandra Bem combined cognitive development theory and social learning theory and theorized that gender roles are predominantly determined by culture and social learning (Response 3). A person's ideas about how males and females should behave in society from the person's "gender schema." Bem developed Gender Schema Theory in reaction to other theorists who proposed that gender roles are in some way innate; that we are born with certain predetermined traits based on our biological gender (ruling out Response 1 and 2). According to Bem, gender roles are not necessarily static over time (Ruling out response 4) as they are altered over time by experience.

1.) Of the following statements, which best describes current contemporary beliefs about development? a. Genes and environment each contribute about equally to development. b. Genes determine the course and pace of development. c. Genes and environment together shape development. d. In most areas of development, genes will be expressed regardless of environmental exposure.

a. Genes and environment together shape development. a. This statement relates to the "nature versus nurture" debate and is consistent with the transactional model, according to which genetic and environmental variables transact continually over time. It is not accurate to state that genes and the environment each contribute about 50% to development (Response 1). Responses 2 and 4 do not address the role of the environment in development, and are untrue.

1.) You are conducting research in a third grade classroom. To fulfill the ethical requirements for informed consent, you should: a. Get consent from the students and teachers b. Get consent from the parents and school c. Get consent from the teachers and school administrators d. Get consent from the parents

a. Get consent from the parents b. Children are not capable of giving consent (i.e., they lack the capacity for consent), and so, when conducting research with children, consent must be obtained from the legal guardian(s), typically the parent(s). Researchers are also required to secure the assent of children, which simply means their agreement to participate. While the teachers and administrators need to approve of the research being conducted in the classroom or school, a researcher does not need to obtain informed consent from either the teachers or the school administrators (ruling out Responses 1, 2 and 3)

1.) Which of the following theorists would most likely be associated with an emic approach? a. Maslow. b. Kohlberg. c. Piaget. d. Gilligan.

a. Gilligan. b. The etic approach takes a universal view of people, and assumes that there are universal principles underlying personality, psychopathology and psychotherapy. The theories of Maslow (Response 1), Kohlberg (Response 2) and Piaget (Response 3) assume that optimum development is the same for all individuals regardless of culture. Gilligan's views tend to be more sensitive to individual differences; specifically, she takes gender differences into consideration. In that sense, Gilligan's perspective seems to be more consistent with the emic approach, which focuses on understanding a person within the context of the specific cultural background.

1.) The most likely medical explanation for trembling, sweating, weight loss, nausea, restlessness, and dizziness, would be: a. Cushing's disease. b. hypothyroidism. c. Grave's disease. d. withdrawal from opioids.

a. Grave's disease. a. Hyperthyroidism (the most common form being Grave's Disease) is characterized by the symptoms listed as well as agitated depression, insomnia, and impaired memory and judgment. Patients with Cushing's Syndrome (Response 1), which involves excessive corticosteroids, experience agitated depression, irritability, emotional lability, memory and concentration problems, and adiposity (swelling and fattening). Hypothyroidism (Response 2) results in weight gain, sluggishness, fatigue, and impaired memory and intellect. Withdrawal from opioids (Response 4) is characterized by flu-like symptoms including muscle aches, diarrhea, nausea and vomiting, fever, insomnia, and dysphoric mood.

1.) Which of the following statements is true regarding gender and leadership? a. No significant differences have been found between the leadership styles of women and men. b. Group members tend to turn to male bosses more commonly than female bosses for information about how to complete a task. c. Most women prefer a female boss while most men prefer a male boss. d. Female bosses tend to be more strict in order to overcome the biases that exist against women and to establish authority.

a. Group members tend to turn to male bosses more commonly than female bosses for information about how to complete a task. a. Differences have been found to exist between the sexes in terms of leadership, ruling out Response 1. Group members typically tend to turn to male bosses for information and to female bosses for support. Both women and men tend to prefer a male boss (Response 3), perhaps because work tasks necessitate a directive leader, a leadership style more common among men. While some women may be strict in order to overcome gender biases (Response 4), strict female bosses tend to be more of an exception than the rule; overall, women tend to be less directive and more supportive.

1.) The findings that in certain cultures clients frequently present with physical complaings rather than psychological problems lends support to the notion of: a. Cultural syndromes b. Idioms of distress c. Higher rates of somatic symptom disorders in some cultures d. Lack of acculturation

a. Idioms of distress a. According to the DSM-5, cultural idioms of distress refer to culturally preferred ways of expressing distress (e.g., with somatic complaints), or "illness languages" that are unique to specific cultures (Response 2, correct answer). Cultural syndromes (Response 1) are cultures of symptoms that tend to occur in specific cultural groups or communities (e.g., ataque de nervios, khyal cap, shenjing shuairuo). Lack of awareness of the tendency for individuals from certain cultures to express emotional distress with physical complains may potentially lead to inaccurate diagnoses of somatic symptom disorders (ruling out Response 3). While the degree of acculturation (Response 4) may affect the nature of symptom presentation, the question was not asking about what has an impact on the rates of use of idioms of distress

1.) What does the Ethics Code say about the care and termination of animals in research? a. Termination of animals in research is prohibited. b. It is unethical to use animals in research. c. If the animal's life must be terminated, it should be done as quickly and as painlessly as possible. d. Termination of animals in research is permissible if the study has "prospective scientific value."

a. If the animal's life must be terminated, it should be done as quickly and as painlessly as possible. a. According to the Ethics Code, when an animal's life is to be terminated, it should be done rapidly, and with an attempt to minimize pain. The Ethics Code does not specifically prohibit the termination of animals in research (Response 1), nor does it prohibit the use of animals in research (Response 2). Note that the notion of "prospective scientific value" applies to subjecting animals to pain or distress, and not specifically to the termination of animals' lives (Response 4).

1.) When would the administration of Ritalin to a child with ADHD be contraindicated? a. If tics are present. b. If the child has a learning disorder. c. If the child has conduct problems. d. If the child has comorbid depression.

a. If tics are present. a. Ritalin would not be the drug of choice for a child with ADHD who also has tics. Because Ritalin is a stimulant, it may exacerbate the tics. Tics are typically treated with a sedating type of medication, such as the antipsychotics Haldol or Orap. Tics are also sometimes treated with the high blood pressure medications Catapres or Tenex

1.) What can be said about the effect of early stimulation on the development of motor skills in infants and preschoolers? a. Regardless of early stimulation, these skills cannot be achieved earlier. b. Infants who experience early stimulation can achieve motor skills earlier than those who do not, however, there are no long-term differences between these groups. c. Infants who experience early stimulation achieve motor skills earlier than those who do not, and these differences are generally maintained. d. Infants who do not experience early stimulation experience lags in motor development.

a. Infants who experience early stimulation can achieve motor skills earlier than those who do not, however, there are no long-term differences between these groups. a. Studies have shown that early motor skills develop as a result of the interaction between maturation and environmental stimulation. While all babies do follow a similar sequence of development, early stimulation can affect the rate at which motor skills are achieved. There do not appear to be long-term differences between those who have had early stimulation and those who have not. Severe deprivation of environmental stimulation tends to have negative effects on motor development that could have long-term consequences.

1.) A patient with specific phobia, blood-injection-injury type is confronted with the object of her fear. The most likely result is: a. Initial sympathetic activation followed by a sudden drop in sympathetic activation b. Prolonged sympathetic activation c. Prolonged parasympathetic activation d. Sympathetic activation that does not differ from an individual who has a different Specific Phobia (e.g., Animal Type)

a. Initial sympathetic activation followed by a sudden drop in sympathetic activation a. In most instances of specific phobia, when exposed to the feared stimulus (e.g., the snake or airplane), the individual experiences prolonged activation of the sympathetic nervous system- blood pressure and heart rate rise when exposed to the feared stimulus (Response 2). However, for individuals with specific phobia, blood-injection-injury type, when confronted with the object of the phobia (e.g, when blood is being drawn for a medical procedure), there is an initial rise in sympathetic activation followed by a sudden drop in sympathetic activation (Response 1); the sudden drop in blood pressure and heart rate may lead to fainting (the heart is not pumping sufficient oxygen to the brain). Specific phobia, blood-injection-injury type usually starts in childhood and commonly runs in families

1.) Research on cognitive changes associated with aging have generally supported which of the following set of findings? a. Decreased attention, intact short-term memory, improved problem solving. b. Intact attention on simple tasks, decreased processing speed, improved verbal abilities. c. Intact fluid abilities, decreased crystallized abilities, difficulty with long-term memory retrieval. d. Decreased problem-solving, decreased cognitive speed, increased cognitive flexibility.

a. Intact attention on simple tasks, decreased processing speed, improved verbal abilities. a. Older adults tend to do well on simple tasks of attention. Their verbal abilities are generally intact or may even have improved. Declines are commonly seen in processing speed. Generally, short-term memory is intact, problem-solving declines (Response 1), and fluid abilities decline while crystallized abilities remain intact (Response 3). Furthermore, there is frequently a reduction in cognitive flexibility (Response 4).

1.) Quasi-experimental designs affect: a. Internal validity b. External validity c. Both internal and external validity d. Neither internal nor external validity

a. Internal validity a. The hallmark of a quasi-experimental design is non-random assignment of subjects. For example, two different treatments designed to improve critical thinking skills are administered to two first grade classes. Since the students are already in pre-existing classrooms, the students are not randomly assigned. Non-random assignment is sometimes referred to as a "selection bias," and it affects internal validity only (Response 1). Internal validity pertains to whether something other than treatment may have caused the results. In this case if one class improves significantly more than another, it would be hard known whether this was actually due to the treatment or due to preexisting differences among the students in the two classrooms. External validity (Response 2) pertains to the generalizability of results. In this case, if significant results were found we would be confident generalizing to other first graders; thus external validity is not threatened by non-random assignment

1.) Motivation that leads to involvement in an activity without environmental reinforcement is called: a. Intrinsic b. Internal c. Extrinsic d. External

a. Intrinsic a. This question is asking about the concept of intrinsic motivation, or doing something simply because one wants to, without any incentives from the environment. Extrinsic motivation (Response 3) describes wanting to engage in an activity because of an external incentive or consequence. The terms internal and external (Responses 2 & 4) typically refer to the concept of locus of control, and not motivation.

1.) Post-concussion syndrome is characterized by which of the following? a. Irritability, fatigue, and headaches b. Loss of consciousness for minutes to hours c. Loss of consciousness for hours to days d. Headaches and numbness on one side of the body

a. Irritability, fatigue, and headaches a. A postconcussion syndrome is characterized by a constellation of somatic and psychological symptoms, the most common of which are irritability, fatigue, headache, and dizziness. Loss of consciousness for minutes to an hour (Response 2) may occur as a result of either a concussion or a contusion (a more severe blow to the head). Loss of consciousness for hours to days (Response 3) describes a coma. Headaches and unilateral numbness (Response 4) can occur after consciousness is regained following a contusion.

1.) As validity approaches the value of 1.0, what happens to the standard error of the estimate? a. It approaches 1.0. b. It approaches 0.0. c. It approaches the standard deviation of the predictor. d. It approaches the standard deviation of the criterion.'

a. It approaches 0.0. a. When a test's validity approaches 1.0, it means that the test is close to being perfectly valid, or being a perfect predictor. In this case, there would be almost no error in estimating, thus the standard error of the estimate would approach 0.0. The formula for the standard error of estimate is Sest = SDy√ 1-rxy2. The range of the standard error of estimate is from a minimum value of 0.0 to a maximum value of SDy. When a test is a perfect predictor (validity = 1.0), the standard error of estimate is 0.0, or, in other words, the test has no error in prediction. Conversely, when a test has no ability to predict (validity = 0.0), the value of the standard error of estimate is equal to the standard deviation of the criterion (SDy) (Response 4).

1.) Which of the following is true about the capacity of working memory? a. It can hold a limited amount of information for an unlimited time. b. It can hold an unlimited amount of information for a limited time. c. It can hold an unlimited amount of information for an unlimited time. d. It can hold a limited amount of information for a limited time.

a. It can hold a limited amount of information for a limited time. b. Working memory, also known as short-term memory (STM), is very limited in both capacity and duration. Short-term memory can hold 7 + 2 items for the duration of up to 30 seconds. Sensory memory (iconic and echoic memory) can hold significantly more information than STM but there are still definite limits to its capacity. The duration of sensory memory is very short, holding visual images for half of a second and auditory images for up to four seconds. Long-term memory can hold an unlimited amount of information for an unlimited time (Response 3).

1.) Which of the following statements is not true about Prozac? a. It causes mild to moderate anticholinergic side effects. b. It can be used to treat obsessions in OCD. c. It has been used to treat eating disorders. d. It exerts its action on serotonergic pathways.

a. It causes mild to moderate anticholinergic side effects. a. Unlike the tricyclic antidepressants (e.g., Elavil, Anafranil, Tofranil), Prozac, and the other SSRIs do not cause anticholinergic side effects. Common side effects of the SSRIs include headaches, nervousness, restlessness, insomnia, gastrointestinal distress, and sexual dysfunction. Prozac has been used to treat OCD (Response 2) and eating disorders (Response 3), in particular, bulimia. Prozac is a selective serotonin reuptake inhibitor, therefore, it works on serotonergic pathways (Response 4).

1.) Which of the following statements about interpersonal therapy (IPT) for depression is inaccurate? a. Its goal is to improve relationships. b. It is commonly combined with medication. c. It is practiced on a short-term basis. d. It emphasizes current conflicts and intrapsychic issues.

a. It emphasizes current conflicts and intrapsychic issues. b. Interpersonal Therapy (IPT) is a form of brief therapy (12 - 16 sessions), which was developed for treating depression. The focus in IPT is on current relationships (Response 1) because it is believed that interpersonal problems are commonly associated with acute depression. IPT is frequently combined with antidepressant medication (Response 2). IPT focuses on current conflicts; developmental and intrapsychic issues are not emphasized (Response 4).

1.) What can be said about a Kappa coefficient of .90? a. It indicates a high degree of interrater reliability. b. It indicates a high degree of reliability after adjustments made with the correction for attenuation formula. c. It indicates good criterion-related validity. d. It suggests high split-half reliability after correction with the Spearman-Brown prophecy formula.

a. It indicates a high degree of interrater reliability. a. The Kappa coefficient is a measure of interrater reliability. Kappa of .90 suggests a high degree of consistency among the ratings of the various raters. The correction for attenuation formula (Response 2) is used to determine how much the criterion-related validity coefficient would increase if the both the predictor (test) and criterion (outcome) were perfectly reliable. The Kappa coefficient is not used for either criterion-related validity or split-half reliability.

1.) What is a trainability test? a. It is used to determine whether someone would benefit from training. b. It is used to determine whether the material learned during training has been transferred to the actual job. c. It is used to determine whether the material presented during training has been absorbed by the trainees. d. It is used to determine what materials to use during training.

a. It is used to determine whether someone would benefit from training. a. A trainability test is used to help determine whether employees would actually benefit from training. A trainability test may be a work sample, e.g., the trainee is given a short period of skill instruction, and is then given a test of work performance. Trainability tests may also involve mini-courses, in which trainees are given a small subset of the course material, and are then given a test to measure their knowledge of the material presented.

1.) Several school age children are playing handball during recess. Which of the following statements might be made if the children are in Piaget's stage of autonomous morality? a. It's okay to change the rules if we all agree. b. It is wrong to break the rules of the game. c. You're cheating, so I'm not going to play anymore. d. You're cheating, so I'm going to tell on you.

a. It's okay to change the rules if we all agree. a. This question is based on Piaget's theory of moral development. Response 1 exemplifies autonomous morality while the other three responses are examples of heteronomous morality. According to Piaget, heteronomous morality is seen in children aged 5 -10, and is characterized by a belief that rules are decided upon by authority figures, cannot be changed, and must be followed without question. The transition to autonomous morality occurs at about age ten. In this stage, children realize that rules are flexible, are agreed upon by others, and can be changed if necessary. Note that Response 1 is the only choice that does not see rules as absolute and inflexible.

1.) Which of the following statements about the serial position effect is accurate? a. Uncompleted tasks are better recalled as compared with completed tasks. b. Items at the beginning of a list are better recalled as compared with items in the middle or at the end of a list. c. Items at the beginning and at the end of a list are better recalled as compared with items in the middle of a list. d. Items at the beginning and in the middle of a list are better recalled as compared with items at the end of a list.

a. Items at the beginning and at the end of a list are better recalled as compared with items in the middle of a list. a. According to the serial position effect, the items at the beginning and the end of a list are remembered better on immediate recall than are those in the middle of the list. This effect occurs because the items at the beginning have been rehearsed the most and reached long-term memory, while those items at the end are still in short-term memory. Response 2 describes what is referred to as a primacy effect with regard to memory. The primacy effect is most likely to occur when the person is given a new task (e.g., reciting the alphabet backwards) immediately following the administration of a list of words. The new task interferes with keeping the final items in short-term memory. Response 4 does not describe a phenomenon that occurs in memory research. Response 1 describes the Zeigarnik effect.

1.) In the first session of structural family therapy, the therapist is most likely to: a. Join the family but not challenge their interactions b. Start to slowly challenge their interactions c. Establish him or herself as the authority d. Begin immediately to unbalance the family system

a. Join the family but not challenge their interactions a. In the first session of family therapy, a structural therapist typically joins the family by observing their dynamics and relating with family members within the style of their interactions. The other responses also describe elements of structural family therapy, however, it is important to note that generally only after the therapist has joined the family or established rapport, will he or she begin to restructure the family system.

1.) Which of the following does Donald Super's Career Rainbow model emphasize? a. Life roles. b. Occupational roles. c. Self-actualization. d. Upper and lower level needs.

a. Life roles. a. A difficult question! Super's Career Rainbow Model emphasizes all of one's life roles, not just occupational roles. You may have been able to eliminate Responses 3 and 4, in that self-actualization is associated with Maslow, and upper and lower level needs are associated with Herzberg's two-factor theory of motivation.

1.) You have been referred a patient by a managed care company. Which of the following factors would be most important to discuss with this patient? a. Co-payments. b. Limits imposed on treatment. c. Limits of confidentiality. d. Nature and modality of treatment.

a. Limits of confidentiality. a. This is a difficult question, in that all of the factors delineated would be important to discuss. The general rule of thumb, however, is that confidentiality issues supersede almost anything else (except Tarasoff, child abuse, etc.). It would be most important for the client to know what the potential limits of confidentiality related to the managed care company might be (e.g., do they require information about diagnosis, progress, etc.).

1.) A researcher may opt to run a MANOVA rather than an ANOVA because: a. MANOVA can accommodate more than one outcome measure. b. MANOVA can accommodate more than one independent variable. c. MANOVA is more powerful than an ANOVA. d. MANOVA can remove the effects of a confound.

a. MANOVA can accommodate more than one outcome measure. a. A key distinction between a MANOVA and an ANOVA is that a MANOVA can, and should be run when there is more than one dependent variable in a research study. A factorial ANOVA can accommodate more than one independent variable (IV), whereas a one-way ANOVA can only accommodate one IV (Response 2). A MANOVA is actually less powerful than an ANOVA (Response 3). In other words, it's easier to find significance if separate ANOVAs are run for each IV, however, this will increase the chance of Type I error. The ANCOVA is used to remove the effects of a confound (Response 4).

1.) Hypertension and diabetes increase the risk for: a. Substance/medication-induced major neurocognitive disorder b. Major neurocognitive disorder due to traumatic brain injury c. Major neurocognitive disorder due to Alzheimer's disease d. Major vascular neurocognitive disorder

a. Major vascular neurocognitive disorder b. According to the DSM-5, major neurocognitive disorder (NCD) is diagnosed when there is significant cognitive decline from a prior level of functioning in one or more domains (complex attention, executive function, learning nad memory, language, perceptual-motor, or social cognition), and the cognitive deficits interfere with independent functioning in everyday activities. Substance/medication-induced major neurocognitive disorder (Response 1) includes neurocognitive impairments that persist beyond intoxication and withdrawal (e.g, from alcohol, inhalants). Major neurocognitive disorder due to traumatic brain injury (Response 2) results from traumatic brain injury in which there is one or more of the following: loss of consciousness, posttraumatic amnesia, disorientation and confusion, neurological signs (e.g., seizures, hemiparesis), and symptoms persist past the acute post-injury period. Major neurocognitive disorder due to Alzheimer's disease (Response 3) involves an insidious onset and gradual progression of impairment, with either evidence of a causative Alzheimer's disease genetic mutation or clear evidence of a decline in memory, learning, and at least one other cognitive domain. Alzheimer's disease is characterized by senile plaques and neurofibrillary tangles which are found throughout the brain. Major vascular neurocognitive disorder (Response 4, correct answer) involves a stepwise pattern of deterioration and a patchy distribution of deficits. Both hypertension and diabetes may contribute to vascular disease

1.) You are a psychologist working in a multi-disciplinary clinic. A 15-year-old patient of yours tells you that he recently engaged in a sexual relationship with a 35-year-old woman who also happens to be a clinic patient. How should you handle this revelation? a. Maintain confidentiality because the alleged perpetrator is a clinic patient b. Make a child abuse report on behalf of your patient, but do not disclose the perpetrator's name so as to maintain her confidentiality c. Make a child abuse report disclosing the names of all parties involved d. Further explore the situation with your client to find out if the relationship was consensual, and file a report if it was non-consensual

a. Make a child abuse report disclosing the names of all parties involved a. The situation presented constitutes sexual abuse of a minor, and must therefore be reported to child abuse authorities. When making the child abuse report, it is important to include the name of not only the victim, but also the alleged perpetrator, in order to protect other minors from being harmed (Response 2). Because of the age difference, a relationship between a 15-year-old and a 35-year-old could not be considered consensual (Response 4).

1.) You are a rural psychologist. A client presents for treatment with a problem you have no prior experience treating. Given the limited psychological resources in the community, you: a. May treat him if you obtain appropriate supervision or consultation b. May treat him only if the situation is a clinical emergency c. May treat him if you have closely related prior training or experience, and make an effort to obtain the necessary competence d. May not treat him under any circumstance

a. May treat him if you have closely related prior training or experience, and make an effort to obtain the necessary competence a. The APA Ethics Code, Section 2.01, discusses competence and notes that when asked to provide services to individuals when other service providers are not available, "psychologists with closely related prior training or experience may provide such services in order to ensure that services are not denied if they make a reasonable effort to obtain the competence required by using relevant research, training, consultation, or study." In non-emergencies, for psychologists with no related prior closely related training or experience, supervision or consultation would not be adequate (ruling out Response 1). In emergencies, when other mental health services are not available, psychologists may provide services even if they do not have the necessary training; however, this is not the only circumstance in which you could treat with no prior experience or competence (ruling out Response 2)

1.) Kuder-Richardson is a: a. Measure of inter-rater reliability b. Method of estimating reliability if the number of items is increased or decreased c. Coefficient of stability d. Measure of internal consistency derived by analyzing all possible split-halves

a. Measure of internal consistency derived by analyzing all possible split-halves b. Kuder-Richardson is a measure of test reliability. Specifically, it measures internal consistency by analyzing all possible split-halves of a test. It resemble Cronbach's coefficient alpha. There are a variety of measures of inter-rater reliability including Pearson r, percentage agreement, Kappa, and Yule's Y (Response 1). The Spearman-Brown prophecy formula is used to estimate reliability if the number of items is decreased or increased (Response 2). When split-half reliability is used, the test is treated as though it were two separate tests with half as many items. Spearman-Brown is therefore needed to correct for the decreased number of items. The coefficient of stability (Response 3) is another name for the test-retest reliability coefficient.

1.) Which of the following is incorrectly matched? a. Melanie Klein: focus on the impact of social relations and cultural factors in determining personality. b. Alfred Adler: disturbance is viewed as a result of a faulty life-style involving a struggle for power. c. Anna Freud: focus on capacities for integration and adaptation and what is interfering with these processes. d. C. G. Jung: views neurosis not as an illness but as striving toward psychological maturity.

a. Melanie Klein: focus on the impact of social relations and cultural factors in determining personality. a. Melanie Klein is an Object Relations theorist, and treatment focus is on recognizing and modifying the impact of problematic early object-relationships. Social relations are a key aspect of this theory, but cultural factors are not. The focus on both social and cultural factors is more characteristic of the neo-Freudians (e.g., Sullivan, Horney), who view psychological disturbance as the result of faulty learning and as consisting of a characterologically maladaptive style of interacting with the environment. Adlerian therapy deals with faulty life-styles that involve a struggle for power (Response 2) as a means of compensating for feelings of inferiority. A focus on capacities for integration and adaptation (Response 3) is characteristic of Ego Psychology (e.g., Anna Freud, Heinz Hartmann). Response 4 describes Jungian therapy. Jung believed that neurosis represents the struggle of people to free themselves from the interference of the archetypes, interference that otherwise impedes progress toward personality integration and fulfillment of potential.

1.) Under which of the following conditions is a group likely to generate the most effective solution to a problem? a. The group has a directive leader. b. Members of the group feel connected with each other. c. Members of the group know each other's strengths and limitations. d. Members of the group are similar in terms of abilities and ideas.

a. Members of the group know each other's strengths and limitations. a. When group members are aware of each other's strengths and weaknesses, they are more likely to make decisions that the group can realistically implement. Responses 1, 2, and 4, all describe situations that could promote groupthink, which would have the ultimate effect of generating a solution that all group members agree upon, but not necessarily a good solution.

1.) The transition from a perspective of "time since birth" to "time left to live" occurs in which of Levinson's stages? a. Age 30 transition. b. Mid-life transition. c. Age 50 transition. d. Late adulthood.

a. Mid-life transition. a. This question refers to Levinson's Seasons of a Man's Life, which is divided into several stages. These stages can be roughly broken into early adulthood, age 30 transition and settling down, middle adulthood, and late adulthood. During the mid-life transition (40-45), the person changes perspective from "time since birth" to "time left to live." The person also begins to develop his life for middle adulthood and begins to relinquish the perspective of early adulthood. As a result, the person is supposed to develop a more realistic view of the world. In the stage of the age 30 transition (Response 1), the person establishes patterns appropriate for adult life. Age 50 transition (Response 3) involves making changes associated with entering this decade of life. Late adulthood (Response 4) involves the developmental tasks associated with this stage of life, such as retirement, death, etc.

1.) One way that a drug can act as an agonist is to: a. Mimic the action of a neurotransmitter b. Block the action of a neurotransmitter c. Reverse the action of a neurotransmitter d. Work with other substances to activate a receptor

a. Mimic the action of a neurotransmitter a. An agonist is a substance t hat binds to a cell receptor and mimics, potentiates, or enhances the pharmacological effect of a neurotransmitter. For example, nicotine is an acetylcholine agonist; nicotine binds to acetylcholine receptor sites. An antagonist is a substance that blocks the action of either a neurotransmitter or an agonist (Response 2). Traditional antipsychotics (e.g., Haldol) are dopamine antagonists. An inverse agonist (Response 3) is a substance that binds to the same cell receptor as an agonist, but produces the opposite effect of the agonist. For example, a substance called Ro15-4513 is an inverse agonist of benzodiazepines. While both Ro15-4513 and the benzodiazepines bind to the same GABA receptor sites, Ro15-4513 has the opposite effect, producing anxiety rather than the calming effect of the benzodiazepines. A co-agonist (Response 4) works with other co-agonists to produce the desired effect together. For example, the co-agonists glutamate and the glycine work together to activate NMDA receptor sites

1.) Which of the following medications should not be prescribed for someone with alcohol use disorder? a. Monoamine oxidase inhibitors b. Selective serotonin reuptake inhibitors c. Neuroleptics d. Opiates

a. Monoamine oxidase inhibitors a. This question is asking specifically which class of drugs should not be prescribed to a patient who has alcohol use disorder. Monoamine oxidase inhibitors (MAOIs) and alcohol can make a lethal combination. Because many beers and wine containe tyramine, even just one drink can sometimes induce a hypertensive crisis in a patient on MAOIs. It is generally safe to prescribe SSRIs (Response 2) as well as neuroleptics/antipsychotics (Response 3) to people with alcohol problems. While prescription opiates or painkillers such as morphine and codeine (Response 4) are generally contraindicated for patients with alcohol use disorder due to the high addition potential, there are still clinical situations (e.g., post surgery) when opiates are prescribed

1.) While a subset of children who are severely abused in childhood develop PTSD, some children do not develop PTSD. This can best be explained by: a. Multifinality b. Equifinality c. Regression to the mean d. The child's age at the time of abuse

a. Multifinality a. Multifinality refers to the phenomenon that similar initial conditions (e.g., severe childhood abuse) may result in different outcomes (i.e., developing PTSD or not developing PTSD). Equifinality (Response 2) refers to the opposite phenomenon, in which different initial conditions lead to the same outcome. If you had trouble choosing between these two options, you might have tried to break down the words to decipher their meaning: "multi" means "many," while "equi" means "equal to." Therefore, you might reason that "multifinality" means "many final outcomes" while "equifinality" means "the final outcome is equal or the same." Meta-analyses of the research looking at the relationship between age at the time of abuse and severity of symptoms have found inconsistent results (Response 4). Regression to the mean (Response 3) is a statistical term that describes the tendency of extreme scores to become less extreme of retesting.

1.) As a method of employee selection, candidates who are hired to work as psychologists for the Department of Veterans Affairs must possess three skills as measured by three different tests. The appropriate method of employee selection in this situation would involve: a. Structural equation modeling b. Multiple cutoff c. Multiple regression d. Discriminant function analysis

a. Multiple cutoff a. This scenario requires the use of a multiple cutoff procedure, in which all three predictor tests are administered to each applicant. In order quality, the applicant must pass each test; in other words, the applicant must reach the cutoff on each test. The multiple cutoff procedure is non-compensatory. Both multiple regression (Response 3) and discriminant function analysis (Response 4) are compensatory procedures; scores on the various predictor tests are weighted and combined, and higher scores can make up for the lower scores. Multiple regression and discriminant analysis would not be appropriate when a candidate must possess all of the required skills. In multiple regression (Response 3), scores on several predictor tests are combined to derive a criterion or outcome score (e.g., score on the EPPP). Discriminant function analysis (Response 4) is a special case of the multiple regression, in which the scores on several predictor tests are combined to derive a criterion or outcome variable that is categorical (e.g., pass the EPPP or fail the EPPP). Structural equation modeling (Response 1) is a sophisticated correlational technique, and is used to determine whether or not a given causal model among a set of variables is accurate

1.) Which of the following would likely be the most important factor in terms of preventing premature termination of therapy by African American clients? a. Race of the therapist b. Client's level of acculturation c. Client's racial identity d. Nature of the client's interactions with the therapist

a. Nature of the client's interactions with the therapist b. The nature of the therapeutic relationship is the single most important factor in preventing premature dropout with African American clients. Factors in the therapist that can negatively impact outcome include racism, color blindness (e.g., not recognizing the unique struggles of being African American), or a paternalistic approach (attributing all problems to racism). Special attention should be given in the first sessions to establishing a positive alliance. It is important for the therapist to be able to accurately distinguish between cultural paranoia (a healthy suspiciousness in response to racism), which is quite common, as opposed to a functional paranoia (e.g., pathology). Generally a more egalitarian therapy relationship will result in better outcome.

1.) Which of the following statements is true regarding the impact of sociocultural factors on the prevalence and presentation of psychopathology? a. The prevalence rates of schizophrenia in different cultures are virtually the same, though the symptom presentation tends to be different and more specific to the culture. b. Depressive disorders tend to manifest similarly across cultures, most commonly including feelings of guilt and worthlessness. c. Non-psychotic disorders are more prevalent among individuals of higher SES. d. Dementia and other cognitive disorders seem to be equally prevalent among persons of low and high SES.

a. Non-psychotic disorders are more prevalent among individuals of higher SES. a. A major epidemiological study found a definite relationship between social class and mental illness. Neurosis was found to be more prevalent among higher SES groups while psychosis was more prevalent among lower SES groups. There remains controversy whether socioeconomic status causes mental illness, mental illness causes socioeconomic status, or diagnostic procedures are inherently biased. Prevalence rates for schizophrenia are also highly variable, and range from .1% in non-Western societies to 1% in Western societies. People from ethnic minority groups are more frequently misdiagnosed (Response 1). Note that while a "core" syndrome appears to exist across cultures, the specific content of the hallucinations and delusions as well as the prevalence of visual, tactile and olfactory hallucinations varies. One of the most notable findings in cross-cultural studies of depression is the significant variation in the symptom presentation (Response 2). Factors associated with lower SES, such as poverty and lower levels of industrialization influence the prevalence rates of diseases affecting the brain, and may increase the rate of dementia, delirium, and other specific cognitive syndromes in persons of lower SES (Response 4).

1.) You have been treating a psychologist in your private practice for the past few months. During a session, he expresses rage toward one of his patients and voices imminent intent to harm this patient. What should you do? a. Maintain confidentiality since this man is your patient. b. Keep this information to yourself since you do not want to breach confidentiality by identifying the name of this man's patient. c. Address the implications of your patient's behavior if he chooses to act on his impulses. d. Notify the police and the intended victim

a. Notify the police and the intended victim b. Your patient (who happens to be a psychologist) has communicated a threat of harm against an identifiable victim, who happens to be one of his patients. This situation meets all the criteria for a Tarasoff situation. You are therefore legally mandated to notify the police as well as the intended victim.

1.) According to Piaget, development characterized by which of the following is most likely to account for separation protest? a. Symbolic representation b. Object permanence c. Irreversibility d. Egocentric thinking

a. Object permanence a. Object permanence develops during the sensorimotor stage and refers to the ability to understand that objects continue to exist independent of involvement with them. Infants become able to maintain a mental image (representation) even when the object is not present. This enhanced capacity has been used to explain the development of separation protest as well as stranger anxiety. Symbolic representation (Response 1), which occurs toward the end of the sensorimotor stage, refers to the child's ability to represent thoughts with symbols. Symbolic representation is critical for language development. Irreversibility (Response 3) occurs in the preoperational stage and refers to the inability to mentally undo an operation. Egocentric thinking (Response 4) also occurs in the preoperational period and describes the child's inability to take the physical perspective of the other person.

1.) Which of the following statements is accurate regarding sleep and older adults? a. Older adults spend less time in Stage 1 sleep b. Older adults require more time to fall asleep c. Older adults spend more time in REM sleep d. Older adults have more frequent awakenings

a. Older adults have more frequent awakenings b. Older adults have more frequent awakenings, which in turn results in diminished total sleep time. In a research project analyzing 65 sleep studies, it was found that compared with teenagers and young adults, healthy middle-aged and older people sleep a half hour to an hour less each night, they wake up a bit more often during the night, and their sleep is lighter. Contrary to expectations, middle-aged and older people do not have more difficulty falling asleep (ruling out Response 2). Sleep researchers have also found that when older adults do suffer from poor sleep, it is typically not because of aging itself, but mostly because of illnesses associated with gaining or the medications used to treat the illnesses. Older adults spend less time in the deep stages of sleep, Stage 3 and Stage 4, as well as in REM sleep (ruling out Response 3). They spend more time in the lighter stages, Stage 1 (ruling out Response 2) and Stage 2

1.) Which of the following statements is true regarding the use of mental health services by minority individuals? a. Overall, minority persons underutilize mental health services b. Overall, minority persons overutilize mental health services c. Minority persons use mental health services about as often as persons of non-minority background d. Minority persons who seek treatment appear to benefit more than persons of non-minority background

a. Overall, minority persons underutilize mental health services a. This is a hard question because the research in this area is inconsistent with the way the question is asked. While the literature clearly suggests that overall, persons of minority background are either underserved (they underutilize treatment) or inappropriately served (dropout rates are significantly higher than those of Caucasians), a large study found that African Americans and Native Americans were overrepresented in community mental health centers, while Hispanics and Asians were underrepresented. Overall, it seems clear that ethnic minorities do not fare well in the mental health system.

1.) Which of the following statements is true regarding communication in cultures considered to be high-context? a. People focus more on how the message is communicated. b. Greater emphasis is placed on choosing the right words to express the message. c. People prefer to sit or stand close to the person they are talking to. d. People focus more on the content of the message.

a. People focus more on how the message is communicated. a. In a high-context communication, the situation and the non-verbal cues significantly affect the meaning of what is verbalized. Responses 2 and 4 both describe a low-context communication, in which the meaning of the communication is based on what is explicitly verbalized. Middle-class White American communication tends to be low-context, whereas the communication of African Americans, Asian Americans, Hispanics, and Native Americans is generally high-context. Proxemics (Response 3) refers to the perception and use of personal space. On the whole, Latin Americans, Africans, African Americans, Arabs, South Americans, and the French prefer to sit or stand closer to someone they are talking to than does the typical middle-class White person

1.) Which of the following statements is most consistent with the tenets of the health belief model? a. People believe that exercise and good nutrition are important for promoting good health. b. People believe that emotional stability is instrumental in maintaining good health. c. People increase their healthy habits if they feel more in control of their health. d. People believe that it is important to have regular visits with physicians in order to maintain good health.

a. People increase their healthy habits if they feel more in control of their health. a. The Health Belief Model is a theory about the use of health care services. The model uses a number of variables to predict who will use health care services, including perception of vulnerability to illness, cost of treatment, benefits of seeking treatment, seriousness of the illness, motivation to maintain health, and the factors that led to considering the risks. The Health Belief Model asserts that people's beliefs about their own health are an important determinant of how they engage in health-related behaviors. If they believe that they can exert control over their own health, then they are more likely to engage in healthy habits such as exercise and good nutrition, and to decrease harmful behaviors, such as overeating, smoking, etc.

1.) The block design and picture arrangement subtests of the WISC-IV measure: a. Working memory b. Processing speed c. Verbal comprehension d. Perceptual reasoning

a. Perceptual reasoning b. Block design and matrix reasoning are both performance subtests. These subtests, along with picture concepts, comprise the perceptual reasoning index. The subtests that comprise the working memory index (Response 1) are digit span and letter-number sequencing. The subtests that form the processing speed index (Response 2) are coding and symbol search. The verbal comprehension index (Response 3) includes vocabulary, comprehension, and similarities

1.) The diathesis-stress model proposes that mental illness is the result of: a. Continuously high levels of stress over an extended period b. Brief periods of high stress occurring at regular intervals over time c. Predisposing bodily causes and strain from external pressures d. Family interactions, especially those that are highly stressful during childhood and adolescence

a. Predisposing bodily causes and strain from external pressures a. The diathesis-stress model posits that mental illness results from the combination of nature and nurture. Nature is the person's biological/genetic vulnerability (diathesis), while nurture refers to the person's life experiences (e.g., stress). The diathesis-stress model is frequently used to explain the development of schizophrenia. Although dysfunctional family interactions are frequently implicated as the stress that precipitates schizophrenia, Response 4 is incomplete because it does not include the biological/genetic vulnerability (diathesis) component of the model. Similarly Response 1 and 2 are incorrect because they do not include a biological vulnerability

1.) A 50-year-old man complains to his general physician that he suffers from premature ejaculation. His physician would be most likely to: a. Prescribe Viagra (sildenafil citrate) b. Prescribe testosterone c. Prescribe Paxil (paroxetine) d. Recommend non-pharmacological treatment, as no medication appears effective in the treatment of premature ejaculation

a. Prescribe Paxil (paroxetine) a. The SSRIs (e.g., Paxil, Prozac, and Zoloft) and the tricyclic antidepressant Anafranil (clomipramine) are used to treat premature ejaculation (ruling out Response 4). Viagra (Response 1) is used to treat problems with erections. Testosterone (Response 2) is typically used for low sexual desire. It will not improve sexual functioning in a man with normal testosterone levels. Non-pharmacological treatment of premature ejaculation (Response 4) include the squeeze technique and the stop-start technique developed by Masters and Johnson

1.) According to Vygotsky, what is involved with working in the zone of proximal development? a. Providing the child with assistance in completing a task. b. Providing the child with assistance at the level the child seems to need it. c. Providing the child with assistance until she can master a task on her own. d. Providing the child with assistance that is appropriate to the child's age.

a. Providing the child with assistance at the level the child seems to need it. a. Scaffolding, or working in the zone of proximal development, involves providing assistance only when the child truly needs it, through a process that combines shaping and modeling. In scaffolding, the adult tunes in to what skills the young child has and what skills are needed for the next phase of growth. Response 1 is too general. Response 3 focuses on providing assistance until the task is mastered, but misses the idea of scaffolding, or working at the child's level. Response 4 focuses on the child's age, which can be misleading (e.g., some six-year-olds can read fluently, while others don't yet know the alphabet).

1.) Aphasia is to speech as apraxia is to: a. Recognition of objects b. Gait c. Poverty of speech d. Purposeful movement

a. Purposeful movement b. Apraxia is a neurological disorder that involves the loss of ability to carry out learned (familiar) movements, despite having the desire and the physical ability to perform the movements. The inability to recognize familiar objects (Response 1) is agnosia. Problems with gait (Response 2) are referred to as ataxia. Poverty of speech (Response 3) is termed alogia

1.) Quality control circles are best characterized by which of the following statements? a. They are concerned with improving working life as a whole b. While they are comprised of small work teams, quality control circles typically affect the whole organization c. Quality control circles typically include members from various departments in the organization d. Quality control circles are concerned with giving workers greater responsibility

a. Quality control circles are concerned with giving workers greater responsibility b. The goal of quality control circles (QCCs) is to improve the finished product and level of production. In contrast, the goal of improving the quality of work life (Response 1) characterizes QWL (quality of work life) programs. The method of QCCs involves increasing workers' responsibility for their work, as well as increasing their participation in decisions affecting the nature of the work. QCCs typically affect only the organizational unit, and not the organization as a whole (Response 2). QCCs consists of 7-10 employees from the same department (Response 3).

1.) A person is able to recite a poem from memory that he heard at a poetry reading two hours ago. This is an example of which type of memory? a. Remote memory. b. Working memory. c. Implicit memory. d. Recent memory.

a. Recent memory. b. Recent memory is an aspect of long-term memory that lasts for up to two weeks. Information recalled after two hours is stored in long-term memory. Remote memory (Response 1) typically refers to memory of information stored for two weeks or more. Working memory, another name for short-term memory (Response 2), stores information for only a short duration of up to 30 seconds. Implicit memory (Response 3), also called procedural memory, involves the recollection of skills and physical operations, such as procedures that can be remembered automatically without conscious awareness (e.g., riding a bicycle).

1.) You are asked by a girl's parents to evaluate her cognitive functioning. You learn that the girl was born in Spain, and that she is bilingual. During the course of the evaluation, you begin to suspect that she may have difficulties with English. You do not speak Spanish, and therefore you bring this up with the parents and suggest that she be evaluated by someone who is Spanish-speaking. However, the parents state that they feel comfortable with you and do not want her to see someone else. What would be the best course of action in this situation? a. Proceed with the evaluation to see if your suspicions are justified; if so, bring this up again with the parents. b. Continue to evaluate her but seek supervision. c. Administer non-verbal measures that do not require knowledge of English. d. Refer her out.

a. Refer her out. b. The best thing to do in this situation would be to continue to discuss your concerns with the parents and to explain to them why a referral to a Spanish-speaking psychologist is needed. However, that is not one of the choices, which leaves referring out (Response 4) as the best answer. It does not make sense to continue to evaluate her to investigate your suspicions (Response 1), especially if you do not speak Spanish. It also does not make sense to seek supervision (Response 2) because you need to be able to effectively communicate with this child during testing, which you may not be able to do because of the language barrier. You may be able to administer non-verbal measures (Response 3) however, such tests may not be appropriate for this evaluation, or their norms may not be applicable.

1.) You are one of a few psychologists who works in a small community. Your best friend's husband requests to see you for therapy because you are the only Christian therapist in town and he is only willing to work with a Christian therapist. Furthermore, he states he is unwilling to travel 50 miles to the next town to seek treatment. How should you respond to this request? a. Agree, as long you believe that your objectivity will not be impaired. b. Agree, as you are the only Christian therapist in town. c. Refuse, if you can ascertain that no harm will come to him if he doesn't receive treatment. d. Refuse, given the nature of your connection with him.

a. Refuse, given the nature of your connection with him. b. In this situation, accepting this man into therapy would constitute a multiple relationship. According to the Ethics Code, multiple relationships are not acceptable if they impair a psychologist's objectivity or if they harm or exploit the other party. Given that this man is the husband of your best friend, it is unlikely that you could remain objective in providing treatment to him. It would therefore be inappropriate to see him in therapy.

1.) A group of very anxious patients is selected based on elevated scores on Spielberger's State-Trait Anxiety Inventory (STAI). Each subject is administered eight sessions of biofeedback assisted relaxation training, and is then measured on the STAI post-treatment. The greatest threat to the validity of this research is: a. Regression to the mean b. History c. Selection d. Instrumentation

a. Regression to the mean a. Regression to the mean is a significant concern when subjects initially score extremely high or low on a measure. Upon retesting, there is a tendency to score in a less extreme range, or closer to average. In this scenario, the subjects might appear as if they had improved due to treatment, when it is simply a result of regression. The best way to manage the threat of regression is with a control group. History (Response 2) refers to specific incidents that intervene between measuring points, either inside or outside of the experimental situation. While history is certainly a potential threat, it is not as great a threat as regression. Selection (Response 3) specifically refers to problems due to non-random assignment, and since there was only one group, selection is not of concern. Instrumentation (Response 4) refers to changes in equipment or observers, and since a valid and objective instrument is being used, this is not likely to be of concern.

1.) A person who believes that people who are white are superior to blacks is in which stage of Helms's model of white racial identity development? a. Reintegration b. Immersion c. Pseudo-independence d. Disintegration

a. Reintegration a. According to Helms, a White person in the U.S. is a member of a majority group that is also a socioeconomically & politically dominant group. Up until the point of contact with Blacks (or other minorities), it is possible for a White person to exist without acknowledging the reality of this dominance, & it is also possible not to deal with a sense of White racial identity. Helms's model of White Racial Identity Development includes 6 stages. The 1st stage, called contact, is entered as soon as a White person encounters the idea or actuality of Black people. At this stage there may be a curiosity, timidity or trepidation about Blacks, & a superficial, inconsistent awareness of being White. In the 2nd stage, disintegration (Response 4), there is a conscious, though conflicted acknowledgment of Whiteness and a questioning of the racial realities (prejudices) the person has been taught to believe. In the 3rd stage, reintegration (Response 1), the person consciously acknowledges a White identity, including the belief that Whites are superior to people of color. In other words, the person in the reintegration stage accepts the belief in White racial superiority and Black inferiority. In the 4th stage, called pseudo-independence (Response 3), the person begins to actively question the proposition that Blacks are innately inferior to Whites. The person acknowledges the responsibility of Whites for racism, and adopts an intellectualized acceptance of White racial identity as well as a curiosity about Blacks. In the 5th stage, immersion (Response 2), the person examines his or her racial identity and the significance of being Black or White. The goal becomes one of changing White people & their attitudes toward Blacks. In the 6th stage, autonomy, the person no longer feels threatened by those of another race, adopts a multicultural perspective, & is receptive to thinking in new ways about culture and race.

1.) A researcher who wants to include the smallest number of predictors in a stepwise multiple regression would be best advised to: a. Include similar predictors b. Make sure the predictors are moderately correlated with the criterion c. Use the correction for attenuation formula d. Replicate the study on a different sample

a. Replicate the study on a different sample b. This is a very advanced statistics question. A multiple regression equation uses two or more variables (independent variables or predictors) to predict one continuous variable (dependent variable or criterion). A step-wise regression does so with no a priori hypotheses about which variables account for the most variance. (A hierarchical regression, in contrast, is conducted consistent with a proposed theory). A step-wise regression statistically determines which variable accounts for the most variance and then enters it into the equation. Replicating the study would help you find the smallest number of predictors. For example, a stepwise regression on an initial sample might produce ten variables that appear to be good predictors. Actually, however, some of those predictors were only successful by chance. Replicating the study on a second sample would remove such predictors and thereby reduce the overall number of predictor variables. You could also approach this question through a process of elimination. Including similar predictors (Response 1) would give rise to multicollinearity, which is a serious problem in regression equations. Our predictions are best when the predictor variables have as low a correlation as possible with each other. While having the predictors moderately correlated with the criterion is a good thing in a regression equation, in and of itself it will not help us reduce the number of predictors (Response 2). The correction for attenuation formula (Response 3) isn't used in multiple regression equations. It is used to estimate how much more valid a test would be if the predictor test and criterion were perfectly reliable.

1.) You have been treating a woman who has a history of childhood physical abuse. She informs you that in the last week she lost her temper with one of her children, and hit him with a belt on his bare bottom. She expresses tremendous guilt and remorse, and swears she will never behave like this again. Which of the following would be your best course of action? a. Report the abuse during the session. b. Work to help your client enhance her coping resources (e.g., mobilizing support, learning stress management techniques, improve parenting skills). c. Work to maintain the therapeutic alliance, address the cycle of violence in your therapy with her, and focus treatment on helping her get over her own abuse, in order to prevent further abuse of her children. d. Make a child abuse report only if the situation recurs.

a. Report the abuse during the session. a. Of all the choices listed, reporting the abuse during the session is the best answer. When a therapist learns that abuse has occurred, or reasonably suspects abuse, he or she must report it. This is the case even if the abuse occurs only one time (Response 4). While it would be important to improve your client's coping resources (Response 2) as well as to address abuse issues in treatment (Response 3), your first step would be to report the abuse

1.) Akathisia refers to: a. Restlessness and dysphoria b. Shuffling gait and tremor c. Apathy and decrease in voluntary movements d. Involuntary movements of the face and limbs

a. Restlessness and dysphoria a. Akathisia, the most common side effect of antipsychotic medications, refers to feelings of dysphoria and an internal sense of restlessness. The restlessness can be expressed as a feeling of the "jitters," tapping of the feet, rocking backward or forward, and shifting weight when standing. All the other responses refer to side effects of antipsychotic medications as well. Response 2 describes parkinsonism. Response 3 describes akinesia. Response 4 refers to tardive dyskinesia.

1.) Which of the following describes banding in personnel selection? a. Persons of somewhat different ethnic origin are banded together within broad categories (e.g., a person from Mexico and one from El Salvador are both considered 'Hispanic'). b. A variety of potentially significant attributes of each job applicant (e.g., work history, education, recommendations) are banded together to form a composite score. c. Groups of interviewers are banded together to form teams, to enhance the reliability and validity of the interview process. d. Scores within a given range are considered equivalent.

a. Scores within a given range are considered equivalent. b. Banding is a process whereby a range of scores obtained by numerous applicants on a test is essentially considered to be the same score. For example, on a test of mechanical aptitude, scores between 91 - 100 are considered to be equivalent for selection purposes.

1.) A woman who is experiencing symptoms of anxiety is seeing a psychologist for short-term therapy. She was referred by her physician after she witnessed a bank robbery in which a clerk was shot and wounded. This is an example of: a. Primary prevention b. Secondary prevention c. Tertiary prevention d. Brief therapy

a. Secondary prevention a. This is an example of crisis intervention, which can also be thought of as secondary prevention. Secondary prevention focuses on early identification and prompt treatment of a disorder with the goal of stopping relatively mild disorders from becoming more serious and prolonged, and thereby reducing prevalence by shortening a disorder's duration. Here, the goal is to prevent the development of chronic anxiety (e.g. PTSD as a result of witnessing the incident). Primary prevention (Response 1) focuses on attempts to prevent the onset or occurrence of a disorder, and thereby reduce its incidence. Examples include alcohol and drug education, Head Start, or vaccinations. Tertiary prevention (Response 3) focuses on reducing the residual effects of a disability or minimizing further negative consequences of an established, serious disorder, for example vocational rehabilitation and day treatment centers for schizophrenics. Brief therapy (Response 4) focuses on pathology and improvement in long-term functioning. Here, the focus is on the crisis, or the woman's reaction to witnessing the robbery.

1.) In the course of therapy, it becomes clear that your patient has become sexually attracted to you. Even after you process this with your patient at length, the attraction intensifies, to the point that it is clearly interfering with treatment. What is the next step that you should take in this situation? a. Seek therapy to explore any countertransference you may be having b. Explain to the patient that sexual feelings are not appropriate, give the boundaries of the therapy relationship c. Seek consultation d. Refer the patient out to another therapist

a. Seek consultation a. Given that you have already processed his attraction extensively, the best course of action at this point is to seek consultation to determine how to handle the case. Seeking your own therapy (Response 1) does not really address the issue of how to handle the treatment process, which is being affected by this patient's attraction. In addition, the question does not mention any countertransference issues that would warrant exploration in your own treatment. While sexual behavior within the therapeutic relationship is inappropriate, illegal, and unethical, sexual feelings in and of themselves are not inappropriate (Response 2). Referring him out (Response 4) would be premature at this point, since you have not yet sought consultation to explore available options.

1.) Of the following, which describes the type of memory that can retain an exact copy of what is seen for a second? a. Sensory memory. b. Eidetic (photographic) memory. c. Echoic memory. d. Short-term memory.

a. Sensory memory. a. Sensory memory stores information as a visual (iconic) or auditory (echoic) image. Although the information is retained in sensory memory for a very brief period of time (up to one second for visual information and up to four seconds for auditory information), this information is retained accurately. Note that this question is asking about what is seen, therefore it is referring to iconic memory, not echoic (Response 3). Response 2 is incorrect because photographic memory refers to the ability to recall information for a substantial period of time, not just one second. Short-term memory (Response 4) typically is not an exact copy and holds 7 + or - 2 bits of information for up to 30 seconds.

1.) A surgeon elects not to perform a serious operation on her father. In considering the Yerkes-Dodson law, what can be said about this surgeon's decision? a. She made the wrong decision because a high level of arousal facilitates optimal task performance. b. She made the wrong decision only if her arousal would have exceeded the optimal level for conducting the operation. c. She made a correct decision since her arousal would have been too low for the operation. d. She made a correct decision because her arousal would have been too high for performing the task.

a. She made a correct decision because her arousal would have been too high for performing the task. b. According to the Yerkes-Dodson law, while a moderate level of arousal or anxiety is optimal for task performance, high and low levels of arousal can decrease performance. Here, it is likely that operating on one's father would create a high level of anxiety and would impair performance; therefore, the surgeon made the correct decision.

1.) Testing-of limits: a. May be used to boost an underperforming examinee's score b. Should typically be done after each subtest c. Should typically be done at the end of a test Should typically be used when an examinee is performing very well

a. Should typically be done at the end of a test a. In testing-of-limits, a test is first administered to a student using the standard test administration procedure. After completing the test, subtests are given again to the student with steps to facilitate performance (e.g., additional cues or structure are provided, time limits are eliminated). Testing-of-limits is done when an evaluator want to gain additional qualitative or descriptive information about an examinee. Implementation of testing-of-limits should be hypothesis driven (i.e., there should be a reason for it). For example, if there is concern that time constraints are significantly affecting an examinee's scores, allowing extra time to complete Block Design items could help ferret that out. Or, to help clarify the cause of poor Arithmetic performance, one might repeat some items while giving the examinee a pencil and paper. Experts in psychological assessment (e.g., Sattler) strongly advise that testing-of-limits take place only after the entire test has been completed. Testing-of-limits after each subtest results in a non-standardized administration (ruling out Response 2). Typically, testing-of-limits is not used when an examinee is already performing well (ruling out Response 4). Testing-of-limits provides only descriptive information and should not be used in any way to alter an examinee's scores (Ruling out Response 1)

1.) A researcher is interested in whether there are differences between four different teaching styles for students of three different SES levels. The F ratio for teaching style (A) = 4.7*. The F ratio for SES (B) = 1.7. The F ratio for the Interaction (AxB) = .9. *= p < .05. It is safe to conclude that: a. Significant differences for teaching style and SES were found b. Significant differences for SES and a significant interaction effect were found c. A significant interaction effect was found d. Significant differences for teaching style were found

a. Significant differences for teaching style were found b. While this data may look complicated, it is actually not that difficult to interpret. In any two way ANOVA, three F ratios are calculated, one for each main effect (e.g., teaching style or SES), and one for the interaction. To interpret an ANOVA for significance you need to look at the F ratios; the F ratio(s) that is/are starred are significant, and the significance level is described (e.g., p<.05). So in this example the only significant F ratio is for teaching style.

1.) Which of the following theories of attraction focuses on costs and benefits? a. Similarity. b. Reciprocity. c. Social exchange. d. Matching.

a. Social exchange. a. Social Exchange Theory states that people are concerned with the costs and benefits of being in a relationship. Each person considers the rewards from the interaction as well as the costs incurred in the interaction. When costs outweigh rewards, social attraction declines. Additionally, the person making a relationship choice is most likely to select a partner with the most favorable reward/cost ratio as compared with other available partners. Similarity hypothesis (Response 1) states that people similar in social background and values tend to form intimate relationships. Reciprocity theory (Response 2) asserts that people tend to like others who like them. Matching hypothesis (Response 4) posits that people of approximately equal physical attractiveness are likely to select each other.

1.) Which of the following is associated with brainstorming in a small group? a. Social loafing b. Social inhibition c. Social dilemmas d. Social facilitation

a. Social inhibition a. Social inhibition refers to inhibition (or worsening) of performance, when a novel or complex task is performed in a group. Social facilitation refers to facilitation (or improvement) in performance, when a simple, well-rehearsed task is performed in a group. Because brainstorming is necessary when there are no simple answers to a problem or situation, inhibition is more likely. Also, keep in mind that the research suggests that people actually generate more ideas and better ones when they brainstorm alone, as opposed to brainstorming in a group. Social loafing (Response 1) occurs when members of a group are evaluated based on the performance of the group as a whole rather than their own individual efforts. In such a situation, the members tend to "loaf," i.e., they don't work as hard as they would if their efforts were being individually scrutinized. In social dilemmas (Response 4), participants are rewarded in the short run for competing rather than cooperating, even though all participants would benefit more in the long run by consistently cooperating.

1.) Which of the following would a behaviorist use to treat depression? a. Social skills training. b. Changing maladaptive perceptions. c. Anger management. d. Thought stopping.

a. Social skills training. a. Behaviorists view depression as a result of insufficient reinforcement from the environment. They therefore help clients increase their repertoire of behaviors, which can then be reinforced by the environment. Out of the choices given, social skills training is the best response. Social skills training helps clients develop appropriate skills for interacting with others, thereby increasing the likelihood of reinforcement. Changing maladaptive perceptions (Response 2) is a cognitive intervention and not a behavioral one. Anger management (Response 3) can be one aspect of social skills training, and it is therefore less complete and not the best answer. Thought stopping (Response 4) may be helpful in reducing negative thoughts but does not serve to maximize opportunities for reinforcement.

1.) A researcher is looking at the effects of treatment for depression over time. Subjects are assigned to one of two treatment groups (cognitive-behavioral therapy or medications), or to an attention only control group. Subjects are measured at the start of treatment, at the conclusion of treatment, and at one year post treatment. To analyze her results, the researcher should use a: a. One way ANOVA b. Three way ANOVA c. Repeated measure ANOVA d. Split plot ANOVA

a. Split plot ANOVA b. This experiment involves a mixed design, consisting of two independent variables (IVs). One IV is the repeated measures variable of time, with three levels, because the same subjects are being measured more than once (at the start of treatment, at the conclusion of treatment, and one year post treatment). The data collected on this variable are therefore correlated. The other independent variable is the between-groups variable of treatment, with three levels (e.g., cognitive-behavioral, medications, and attention only). The data collected on this variable are therefore independent. The split-plot ANOVA is the appropriate statistic in this situation. A one-way ANOVA (Response 1) can't be used when there are two IVs. A three-way ANOVA (Response 2) is used when there are three IVs, which is not the case here. In a repeated measures ANOVA, the data on all of the IVs are correlated.

1.) According to most research, parental attention to which of the following senses plays the most critical role in healthy infant development? a. Tactile. b. Gustatory. c. Auditory. d. Visual.

a. Tactile. a. Harlow's studies on contact comfort demonstrated the importance of pleasurable tactile sensations as a key contributor to attachment behavior. Contact comfort appears to be equally if not more important between human mothers and their infants.

1.) You are working in an inpatient treatment facility for adolescents. In the course of your evaluation of a 17-year-old girl, you become aware that her physician appears to have misdiagnosed her condition and is prescribing a medication which is actually exacerbating her symptoms. How should you address this situation? a. Notify the girl immediately and encourage her to follow up to ensure that she gets appropriate treatment b. Notify the girl's parents immediately and encourage them to follow up to ensure that she gets appropriate treatment c. Talk to the treating physician immediately, with appropriate releases d. Refer the girl to another physician for a second opinion, to ensure that she gets appropriate treatment

a. Talk to the treating physician immediately, with appropriate releases a. In this situation, it would be best to initially speak with the treating physician to clarify the situation (the physician's understanding of the girl's problems and why the specific medications have been prescribed), as well as your concerns. Referring the girl for a second opinion (Response 4) would be premature without first contacting her treating physician. While it would be appropriate to share your concerns with the girl's parents (Response 2), encouraging them to follow up bypasses your obligation to first attempt to collaborate with the treating physician. It would be inappropriate to only share your concerns with the girl (Response 1), as she is a minor

1.) The best friend of your seven-year-old daughter just died in a car accident. What does the research suggest would be the best way to handle the situation? a. It would be better not to tell your child that her friend died, because it could unnecessarily fuel fears about her own death and future death of family members. b. It would not be of benefit to tell your child that her friend died, because children this age cannot cognitively comprehend the meaning of death. c. Whether or not to tell your child that her friend died should depend on your child's level of emotional maturity. d. Tell your child that her friend died, because an open, honest discussion will contribute to her emotional well-being.

a. Tell your child that her friend died, because an open, honest discussion will contribute to her emotional well-being. b. The research suggests that most children arrive at an adult-like understanding of death by age seven. This understanding includes the ideas that death is permanent (dead things cannot be brought back to life), universal (all living things eventually die), and nonfunctional (all functions including thought, movement, etc., cease at death). Although parents often worry that discussing death with children will fuel their fears, this is not the case. Instead, children who have a good grasp of the facts of death have an easier time accepting it. Open, honest discussions contribute both to cognitive appreciation of the concept of death, and to the child's well-being. This is also the case when the young child is faced with his or her own impending death (e.g., from a life threatening illness).

1.) Which brain structures play a key role in aggression? a. The amygdala and hypothalamus b. The amygdala and hippocampus c. The thalamus and hypothalamus d. The amygdala and thalamus

a. The amygdala and hypothalamus a. Research on aggression in both humans and animals has implicated both the amygdala (part of the limbic system) and hypothalamus (Response 1) along with the prefrontal cortex. In contrast, the thalamus relays sensory information to various parts of the cerebral cortex and relays motor signals from the cerebral cortex, while the hippocampus plays an important role in consolidation of long term memory

1.) Which of the following statements about hypnosis is not accurate? a. The hypnotic state constitutes a change in consciousness. b. There are tests that can indicate the degree to which someone can be hypnotized. c. The extent to which someone can be hypnotized is related to his or her psychological well-being. d. Hypnosis involves dissociation.

a. The extent to which someone can be hypnotized is related to his or her psychological well-being. a. Hypnosis is described as an alteration in consciousness (Response 1), a dissociated state (Response 4), or a subjective experiential change. The degree to which someone can be hypnotized is not related to level of emotional disturbance (Response 3). There are tests available that can indicate the extent to which someone can be hypnotized (Response 2).

1.) You are treating a six-year-old girl in individual therapy following the divorce of her parents. Occasionally, you have family sessions with her father, who is the custodial parent. You notice that after spending a week with her mother, the girl seems high-strung and clings to her father. Which of the following statements best describes your understanding of this situation? a. The girl's presentation is consistent with abuse. b. The girl's presentation is consistent with a separation-anxiety disorder. c. The girl's presentation is probably a normal response. d. The girl's presentation indicates that she is having difficulty with the divorce and custody arrangements, and it would therefore be advisable to limit visits with her mother to briefer periods of time.

a. The girl's presentation is probably a normal response. a. Out of the options listed, this is the best response. It can be typical of some children of divorced parents to exhibit behavioral disturbances after separation from the custodial parent. While it may be possible that this child is being abused, no information is provided in the question to suggest that abuse is actually occurring. It would therefore be too premature to suspect abuse by either parent at this point without knowing more about the family dynamics (Response 1). The essential feature of separation anxiety disorder is excessive anxiety concerning separation from home or from primary attachment figures, lasting for a period of at least four weeks. This is not the case here (Response 2). Finally, the girl seems to be having a normal reaction to a difficult situation. It would be premature to change the visitation schedule at this point (Response 4).

1.) According to Cross's Black Racial Identity Development Model what is the hallmark of the Internalization stage? a. The individual is not longer conflicted and identifies as just another human being b. The individual experiences his or her Black identity as positive, while tolerating and respecting differences in whites c. The individual experiences an increased desire to become more connected to a black identity d. The individual idealizes black culture

a. The individual experiences his or her Black identity as positive, while tolerating and respecting differences in whites a. According to Cross, Black Racial Identity Development consists of several stages. In the first stage, Preencounter, the individual depends on White society for approval, and the person's has a negative attitude toward Black racial identity. The person is preoccupied with how to be perceived as "just another human being" (Response 1). While resolution of conflict, the first half of Response 1, accurately describes the Internalization stage, the second half of Response 1 spoils the answer. In the second stage, Encounter, the individual experiences confusion about the meaning and significance of race, and experiences an increased desire to become more connected to a Black identity (Response 3). In the third stage, Immersion-Emersion, the individual idealizes and is absorbed in the Black experience and rejects the White world (Response 4). In the fourth stage, Internalization, the individual recognizes that both Blacks and Whites have strengths and weaknesses, and views his or her Black identity as positive, while respecting differences in Whites (Response 2)

1.) Which of the following describes the most effective form of flooding? a. The patient is exposed to the feared stimulus for a prolonged period. b. The patient is exposed to the feared stimulus for repeated, intermittent periods. c. The patient is exposed to the feared stimulus when the fear is excessively heightened. d. The patient is exposed to the feared stimulus for a brief period until the fear is excessively heightened.

a. The patient is exposed to the feared stimulus for a prolonged period. a. Flooding is most effective when the patient receives massed exposure, or exposure for a substantial period of time, until the fear is diminished. Intermittent periods of exposure (Response 2) may not be effective, and it has even been argued that brief exposure to the feared stimulus can intensify a phobia (Response 4). Response 3 is rather vague, and does not indicate the amount of exposure involved, which is considered a crucial aspect of the effectiveness of flooding.

1.) Which of the following statements typifies a person's reaction to an event according to two-factor theory? a. The person experiences autonomic arousal, and then has an emotional reaction. b. The person experiences autonomic arousal and an emotional reaction to the event simultaneously. c. The person experiences autonomic arousal, then cognitively interprets the arousal, and then experiences an emotion. d. The person experiences autonomic arousal, then experiences an emotion, and then cognitively interprets the emotion.

a. The person experiences autonomic arousal, then cognitively interprets the arousal, and then experiences an emotion. a. Schacter's Two-Factor Theory of Emotion states that after perceiving an event, we initially have a physical or autonomic reaction. Next, we then cognitively appraise the situation, and then experience an emotion. Essentially, we look to external rather than internal cues to differentiate and label our emotions. For example, a person experiences palpitations and sweating; if, upon cognitive evaluation, the person realizes that she is having this reaction to a surprise party, she feels excitement. However, if she cognitively appraises the situation and realizes that there is an intruder in the home, she experiences fear. According to the James-Lange Theory (Response 1), one first has an autonomic or physiological reaction, and then an interpretation of this physiological reaction, which becomes the emotional reaction. In other words, physiological changes cause emotions (e.g., "I'm trembling so I must be afraid."). Response 2 describes the Canon-Bard Theory of Emotions, which states that upon perceiving an event, we experience a physiological reaction and an emotional reaction at the same time. This theory states that when an event is encountered, messages are sent simultaneously to the hypothalamus (responsible for physiologically arousing the body) and to the limbic system (causing the subjective experience of fear). For example, upon encountering a bear, one feels fear (emotional reaction), and rapid heart-beat and trembling (physical reaction) simultaneously. Response 4 is incorrect in terms of the sequence described by Two-Factor Theory.

1.) Second authorship on a paper would be least appropriate in which of the following situations? a. The person performed data analysis and interpretation. b. The person contributed to the development of the underlying concepts and structure of the project. c. The person was involved in data collection. d. The person was involved in developing the design of the study and selecting subjects.

a. The person was involved in data collection. a. Of all the choices, data collection is the least significant contribution, e.g., a person could hire someone to hand out questionnaires to subjects as a means of gathering data. Developing the underlying concepts (Response 2) and the design of the study (Response 4), as well as data analysis and interpretation (Response 1), are much more significant contributions, that could warrant second authorship.

1.) In order to successfully sue a psychologist, the patient must prove all of the following, except: a. The psychologist provided negligent treatment b. The psychologist provided treatment with malevolent intent c. The psychologist's treatment caused harm to the patient d. The psychologist had an obligation to provide care for the patient

a. The psychologist provided treatment with malevolent intent a. Several elements must be present in a malpractice lawsuit: an established psychotherapist-patient relationship, which entails a duty to provide care for the patient (Response 4); negligent treatment (Response 1); and harm to the patient as a result of treatment (Response 3). Malevolent intent (Response 2) is not necessary for a malpractice suit

1.) Which of the following statements about immigration is most accurate? a. Persons who are assimilated into the dominant culture are better off psychologically than those who are acculturated, because they experience less conflict. b. There are higher rates of psychiatric hospitalization for immigrants. c. The rate of acculturation for each family member largely depends on each member's values and worldviews. d. Psychiatric problems tend to arise shortly after immigration.

a. There are higher rates of psychiatric hospitalization for immigrants. a. Studies have demonstrated higher rates of psychiatric hospitalization in the U.S. for immigrant groups, especially young men. Overall, research has shown that individuals who are well acculturated show the least stress and the best overall adjustment compared to those who assimilate (Response 1) and to those who refuse to adapt to the dominant culture altogether. While acculturation is certainly influenced by values and worldviews (Response 3), the age of family members plays a more important role in the rate of acculturation, with children and teenagers acculturating more quickly than adults. This differing rate of acculturation often results in a clash of cultural values between the generations. For immigrants, psychiatric problems tend to peak between one year and three years after arrival into the new culture (Response 4).

1.) Which of the following statements is accurate regarding the relationship of age and satisfaction with one's work? a. There is no consistent relationship between age and job satisfaction b. There is a positive relationship between age and job satisfaction c. There is a negative relationship between age and job satisfaction d. The relationship between age and job satisfaction has not been thoroughly investigated by researchers

a. There is a positive relationship between age and job satisfaction a. There is a positive relationship between age and job satisfaction. The older people are, the higher their job satisfaction tends to be

1.) Which of the following complaints is most frequently cited by interns against supervisors? a. Sexual harassment. b. Untimely feedback. c. Lack of competence. d. Inadequate supervision.

a. Untimely feedback. a. Untimely feedback is the most frequently cited complaint against supervisors

1.) Based on Herzberg's two-factor theory, which of the following interventions would be most effective in terms of reducing boredom and increasing motivation in a group of factory workers? a. Increasing salary b. Vertically expanding job responsibilities c. Horizontally expanding job responsibilities d. Implementing job enlargement

a. Vertically expanding job responsibilities a. A vertical expansion of responsibilities, or giving someone higher-level duties, describes job enrichment. Job enrichment increases employees' autonomy, authority and freedom, which are all upper-level needs. According to Herzberg, motivation increases when upper-level needs are met. Overall, job enrichment tends to result in increased satisfaction and performance, decreased turnover and decreased absenteeism. Job enlargement (Response 4) involves simply expanding the variety of tasks without increasing responsibility. Horizontal expansion of responsibilities (Response 3) describes job enlargement. Pay (Response 1) is believed to be a hygiene factor (lower-level need) in Herzberg's theory. Lower-level needs result in dissatisfaction when they are not met, but they are not believed to increase satisfaction or motivation when they are met. Certainly, increasing pay would have no impact on reducing boredom.

1.) Which one of the following theories of leadership addresses decision-making and the continuum of leader behavior, from autocratic to complete participation? a. Vroom and Yetton's normative theory. b. House's path-goal theory. c. Fiedler's contingency theory. d. Graen's vertical dyadic linkage theory.

a. Vroom and Yetton's normative theory. a. Vroom and Yetton's Normative Model states that there are five styles of leader behavior that can be placed on a continuum, ranging from autocratic (decisions made entirely by the leader) to complete participation (decisions made through consensus). The most effective style of leadership in any given situation depends upon the importance of the decision, the time needed to reach the decision, and the extent to which the decision is accepted by the employees. House's Path-Goal Theory (Response 2) deals with the ways in which leaders can help employees achieve their goals, using one of four styles: directive leadership, supportive leadership, participative leadership, and achievement-oriented leadership. The most effective leadership style is dependent on the subordinates and the situation. According to Fiedler's Contingency Theory (Response 3), the effectiveness of the style of leadership is based on the type of leader and the nature of the situation. A leader is classified as either task or relationship-oriented. A task-oriented leader is one who is most effective in highly favorable or highly unfavorable situations. A relationship-oriented leader is most effective in moderately favorable situations. Graen's leader-member exchange theory (Response 4) focuses on the impact of the leader-subordinate relationship on the leadership process. In this theory, employees are classified as in-group (those who are viewed as competent and motivated), and out-group (considered incompetent and unmotivated). Leadership style is classified as 'supervisory,' which is based on formal authority, and 'leadership' in which the leader exerts influence through persuasion. Leaders use the 'leadership' approach with in-group subordinates, and use the 'supervisory' approach with out-group employees.

1.) A defendant with schizophrenia is on trial for indecent exposure. He is deemed incompetent to stand trial. Consequently, the defendant: a. Will never be allowed to stand trial b. Will be treated and re-evaluated for competency to stand trial within 180 days c. Will be tried if he regains competence May be treated to help him regain competence

a. Will be tried if he regains competence a. Incompetence to stand trial refers to a determination that a defendant is unable to understand the nature and consequences of the proceedings against him or to assist properly in his defense. The law dictates that a person who is deemed incompetent to stand trial must receive psychiatric treatment (typically either inpatient psychiatric treatment or psychiatric treatment while in county jail) aimed specifically at enabling the person to stand trial with as little delay as possible (ruling out Response 4 which suggest treatment is optional). Once the psychiatric staff believes the defendant has become competent, the court will hold a hearing. If the defendant is found competent, the court will then order the defendant's release if the defendant is in an inpatient psychiatric treatment facility, and set a date for trial. There is no requirement that the defendant be reevaluated within 180 days of treatment (ruling out Response 2)

1.) An elementary school student has just learned the colors of the rainbow with the acronym ROY G BIV. This is an example of which memory enhancing strategy? a. The method of loci. b. The pegword system. c. Word association. d. Substitute word system

a. Word association. a. Word association involves forming words or sentences with the first letter of the items being memorized (acronyms), or making up stories that connect the items together. The method of loci (Response 1) involves first forming a mental image of a walk through a certain physical location, such as a house. The person then forms a visual image of the words on a list, putting each in a specific place as he or she mentally walks through the house. For example, a person trying to recall a list of grocery items may initially picture bacon on the front door, bread in the kitchen, and meat in the dining room. The pegword system (Response 2) involves first memorizing a set of ten visual images that can later be pegs on which to hang ideas, e.g., one is bun, two is shoe, three is tree. Then the person associates each image with what is to be remembered. For example, if the person is trying to remember a list of grocery items, he may associate soda with hamburger buns, or talcum powder with shoe, etc. In the substitute word technique (Response 4), the person breaks down the word to be remembered into parts, and substitutes words that are more familiar and can be visualized. For example, a person who wants to memorize the word catabolize would break it into cat, a ball, eyes, and then form a visual image of these words.

1.) You have started seeing a patient whose therapy is being covered by her insurance company. After your initial session, you diagnose this woman with major depressive disorder and narcissistic personality disorder. Which of the following describes your best course of action in terms of reporting her diagnoses to the insurance company? a. You should report both diagnoses to the insurance company b. You should not report her diagnoses to the insurance company as that would breach confidentiality c. You should report her diagnosis of major depressive disorder to the insurance company d. You should report both diagnoses to the insurance company if you believe that her narcissistic personality disorder contributed to her major depressive disorder

a. You should report both diagnoses to the insurance company a. Since you have diagnosed this woman with major depression and narcissistic personality disorder, it is important that you disclose both diagnoses to the insurance company. Altering or omitting diagnoses when reporting to an insurance company constitutes insurance fraud.

1.) You have a chemistry exam scheduled in two days. You studied chemistry yesterday and most of today. If you study physics tomorrow, what is most likely to happen when you take your exam? a. You will experience retroactive interference for chemistry. b. You will experience proactive interference for chemistry. c. You will experience decay of your knowledge of chemistry. d. You will experience problems with consolidation of your chemistry knowledge.

a. You will experience retroactive interference for chemistry. a. If you first study chemistry and then physics, it is likely that your knowledge of physics will interfere with your recall of chemistry. This describes the process of retroactive interference, in which recently learned information interferes with the recall of previously learned information. Proactive interference occurs when previously learned information interferes with the recall of recently learned information. This rules out Response 2. Decay theory (Response 3) states that memory deteriorates with the passage of time. This notion does not apply here, since the question is concerned with how studying physics is going to affect recall of chemistry, and not simply the passage of time. Problems with consolidation (Response 4) involve difficulty getting material from short-term into long-term memory. Problems with consolidation are more likely to be due to lack of rehearsal or inadequate depth of processing.

1.) Privilege is: a. both a legal and an ethical issue, and it is held by the client. b. a legal issue, and it is held by the client and the court. c. a legal issue, and it is held by the client. d. a legal issue, and it is held jointly by the client and therapist.

a. a legal issue, and it is held by the client. a. Privilege is a legal issue. It refers to the patient's right not to have confidential (i.e., privileged) information revealed in legal proceedings. It is thus narrower than the ethical concept of confidentiality. Under most circumstances, the patient is the "holder of privilege," and information transmitted during therapy is considered privileged information. The court has the ability to waive privilege (Response 2), however, the court is not the holder of privilege. Exceptions to privilege generally include situations in which patients are suicidal, homicidal, have perpetrated or been the victim of abuse, etc. In legal proceedings, the therapist may assert privilege on the patient's behalf (Response 4), but the therapist is not the holder of privilege. The most common occasions when patients are not holders of privilege include patients who are minors (in which case the parents are usually the holder of privilege), and adults who lack legal capacity (the legal guardian then holds privilege).

1.) Holland's Self-Directed Search questionnaire measures people on six themes or personality types, associated with particular families of occupations. The theory behind the Self-Directed Search is that: a. better outcomes are obtained when people make their own career decisions, rather than find themselves influenced by external sources. b. congruent occupations are those that either match the predominant theme, or match the theme directly opposite of the predominant theme. c. scores on the six themes can be useful predictors of career success. d. a match between a person's personality and occupation results in job satisfaction.

a. a match between a person's personality and occupation results in job satisfaction. b. Studies have generally supported Holland's Personality-Job Fit Theory. Holland's theory proposes that a good fit between personality type and occupation results in job satisfaction. The logic behind the Self-Directed Search is that by matching personality types to congruent occupations, people will bring appropriate interests and skills to a job's demands. A good match should ensure success on the job, which then brings about a high level of job satisfaction. For example, someone who scores highest on the conventional theme will tend to be conforming, efficient, practical, unimaginative and inflexible. Congruent occupations would include accountant, bank teller, file clerk, or corporate manager. Scores on the themes, in and of themselves, do not predict job success (Response 3). Congruent occupations match the predominant theme, but will not match the theme directly opposite, which is most dissimilar (Response 2). While Response 1 may be a correct statement, it is not the theory behind the Self-Directed Search.

1.) A woman feels throbbing on one side of the head that intensifies during activity. She is most likely experiencing: a. a muscle contraction headache. b. sinusitis. c. a migraine headache. d. a cluster headache.

a. a migraine headache. a. Migraine headaches, a type of vascular headache, are characterized by unilateral pain (pain on only one side of the head), that is pulsating, of moderate to severe intensity, aggravated by routine physical activity, and associated with nausea, photophobia (light sensitivity) and phonophobia (sensitivity to noise). The pain tends to be located in the frontal and temporal areas. While "common migraines" do not involve prodromal symptoms, "classic migraines" (found in about 30% of migraine sufferers) include visual auras and sensory disturbances (e.g., hemiparesis). Migraines are much more common in females. Muscle contraction headaches (Response 1), commonly known as tension headaches, are bilateral in location, and are characterized by a dull and steady tightness or pressure around the head, with pain of mild to moderate intensity. Sinusitis (Response 2) results from inflammation of the sinuses. In addition to nasal congestion, sinusitis can involve headaches, which may be described as pain over the eyes, behind the eyes, or above the eyebrows. Cluster headaches (Response 4), a type of vascular headache, typically involve unilateral pain in the orbital (around the eye) or frontal regions. These headaches, which can be precipitated by alcohol ingestion, are more frequent in men, usually begin during sleep, and recur over a period of days or weeks causing closely spaced attacks of intense pain, followed by remissions that can last weeks, months or even years. The pain is frequently described as piercing.

1.) A person has been offered a job. She is excited by the financial opportunity yet concerned about the long hours the job will require. She is experiencing: a. an approach-approach conflict. b. an approach-avoidance conflict. c. an avoidance-avoidance conflict. d. no conflict.

a. an approach-avoidance conflict. a. This is an example of an approach-avoidance conflict, in which a person is both drawn to a goal and repelled by it at the same time. An approach-approach conflict (Response 1) involves being drawn toward two equally attractive alternatives. In an avoidance-avoidance conflict (Response 3), the person must choose between two equally undesirable alternatives.

1.) A 12-month-old infant exposed to the Strange Situation Procedure responds to the mother's return with initial enthusiasm, immediately followed by detachment. This child is most likely: a. anxious-avoidant. b. anxious-ambivalent. c. insecurely attached. d. slow-to-warm-up.

a. anxious-ambivalent. a. This is characteristic of an anxious-ambivalent infant. This type of infant becomes upset when the mother leaves the room. When she returns, the infant seeks close contact while at the same time angrily squirming to get away. These caregivers have been characterized as inconsistent, and insensitive. Anxious-avoidant infants (Response 1) do not seek closeness and contact with the mother, treat the mother like a stranger, rarely cry when she leaves the room, and ignore her on her return. Insecure attachment includes both the anxious-avoidant and the anxious-ambivalent types and is therefore less specific than Response 2. Slow-to-warm-up (Response 4) describes a type of infant temperament not a type of attachment. Slow-to-warm-up infants are mild in responding, somewhat shy and withdrawn, and need time to adjust to new experiences or people.

1.) Oversensitivity to rejection and criticism, and social inhibition are characteristic of: a. borderline personality disorder. b. avoidant personality disorder. c. dependent personality disorder. d. narcissistic personality disorder.

a. avoidant personality disorder. a. These symptoms are characteristic of someone with an avoidant personality disorder. While a person with a borderline personality disorder (Response 1) can be oversensitive to rejection and criticism, the hallmarks of this disorder include instability of mood, interpersonal relationships, and self-image, as well as significant impulsivity. Dependent personality disorder (Response 3) involves a pervasive and excessive need to be taken care of, resulting in clinging and submissive behaviors, difficulty making decisions, and fears of separation. Although dependent personality disorder can also involve oversensitivity, this oversensitivity is not the primary focus as it is in avoidant personality disorder. Narcissistic personality disorder (Response 4) is characterized by a pattern of grandiosity, need for admiration, and lack of empathy. Narcissistic personality disorder can also involve oversensitivity to criticism, though the focus in this disorder is on protecting one's grandiose image. Further, persons with this disorder seek admiration, whereas individuals with avoidant personality disorder are socially inhibited.

1.) At a rally three months before an important election, a political candidate would probably want to speak: a. before all the other candidates. b. after all the other candidates. c. amid the other candidates. d. in any order, as long as the audience is of at least moderate intelligence.

a. before all the other candidates. a. When a speaker wants to influence others and there is a long gap between the speech and the actual event, such as an election, the best chance of being remembered occurs if the speaker goes first. This is known as the primacy effect. If the event is coming up in the immediate future, e.g., within the next few days, the speaker should opt to speak last in order to be remembered by the audience (Response 2). This is known as the recency effect.

1.) You are the owner of a restaurant and want to make the atmosphere more appealing by adding soft lights and warm colors. If you do so, this will probably have the effect of: a. causing normal weight people to become thinner. b. causing overweight people to become more overweight. c. causing normal weight people to become more overweight. d. causing overweight people to become thinner.

a. causing overweight people to become more overweight. a. This question is addressing the role of external cues in overeating and obesity. In this situation, making the atmosphere most appealing would have the effect of causing overweight people, who are more susceptible to external cues, to eat more and therefore become more overweight.

1.) Jung described the archetypes as primordial images and ideas common to all members of the race from the beginning of life. Jung believed that archetypes are aspects of the: a. id. b. ego. c. personal unconscious. d. collective unconscious.

a. collective unconscious. b. Freud divided the structure of personality into three parts consisting of the id, ego, and superego. He referred only to an unconscious that was personal. Jung added the notion of a collective unconscious, aspects within our unconscious that are common to all of us because they are transmitted from generation to generation.

1.) Meta-analysis involves: a. combining the results of different studies into an effect size while assigning greater weight to studies that are of better quality. b. reanalyzing data from different studies and combining the results into an effect size. c. comparing different studies and calculating an effect size without taking into consideration how each study was conducted. d. combining the results of studies into an effect size.

a. combining the results of studies into an effect size. b. Meta-analysis is a statistical procedure that is used to look at "average" outcome over a broad range of studies (e.g., whether psychotherapy is more effective than no treatment). Meta-analysis involves selecting relevant studies and combining the results to calculate an effect size. Before conducting a meta-analysis, certain criteria are established for the studies to be included in the analysis (e.g., based on the type of design of the study, or how each study was conducted) (Response 3). Meta-analysis, however, does not assign any greater weight to studies that are of better quality (Response 1). Also, meta-analysis does not reanalyze the original data from each study (Response 2), but simply combines the results of the studies into an effect size.

1.) The trait that has been associated with successful job performance across all occupations is considered to be: a. emotional stability. b. agreeableness. c. conscientiousness. d. extroversion.

a. conscientiousness. a. These four traits are derived from the Five-Factor Theory of Personality (Costa and McCrae). Of the five factors, conscientiousness is most consistently associated with job success, and it includes being responsible, self-disciplined, organized, and achievement-oriented. Emotional stability (Response 1) describes people who are not neurotic; instead they are even-tempered, calm, and relaxed. Agreeableness (Response 2) encompasses such qualities as being good-natured, gentle, and cooperative. Extroversion (Response 4) characterizes people who are sociable, outgoing, talkative, fun-loving and affectionate. The last of the five factors, not listed in the responses, is openness to experience. The five factors make up the acronym OCEAN (openness to experience, conscientiousness, extroversion, agreeableness, and neuroticism).

1.) To increase reliability, you could: a. increase subject homogeneity and increase the number of test items. b. decrease subject homogeneity and decrease the number of test items. c. increase subject homogeneity and decrease the number of test items. d. decrease subject homogeneity and increase the number of test items.

a. decrease subject homogeneity and increase the number of test items. b. Two of the factors that affect reliability are the number of items and subject heterogeneity. As a rule, reliability increases as the number of items increases (eliminating Responses 2 & 3). Furthermore, reliability increases when subjects are more heterogeneous, or less homogeneous, because this results in an unrestricted range of scores that are then correlated.

1.) A PET scan of the brain of someone with schizophrenia would show: a. increased metabolic rates in the frontal lobes. b. decreased metabolic rates in the frontal lobes. c. increased metabolic rates in the temporal lobes. d. decreased metabolic rates in the temporal lobes.

a. decreased metabolic rates in the frontal lobes. a. PET scans of brains of persons with schizophrenia show decreased activity in the frontal lobes. If you consider the deficits involved in this disorder, such as difficulty with executive functioning, poor attention and concentration, concrete thinking, poor judgment and insight, you will recognize that these are functions of the frontal lobes. Therefore, PET scans would show that this area is less active in schizophrenia.

1.) A 76-year-old man who just came out of surgery exhibits visual hallucinations, does not recognize his relatives, and cannot recall why he is in the hospital. He is most likely experiencing: a. anterograde amnesia. b. retrograde amnesia. c. dementia. d. delirium.

a. delirium. b. These symptoms are characteristic of delirium. Symptoms of delirium include a change in cognitive abilities (e.g., the inability to recognize familiar people), perceptual disturbances (e.g., hallucinations), as well as a disturbance (clouding) of consciousness, in which the person is less aware of the environment. These symptoms have rapid onset, and are brief and fluctuating. Delirium is most common in the elderly and children, due to post-operative states, as well as infections, metabolic disorders (hypoglycemia, renal disease, etc.), and intoxication/withdrawal. Dementia (Response 4) is characterized by impairment in short-term memory or long-term memory, as well as one of the following four conditions: aphasia (speech impairment), apraxia (inability to carry out tasks), agnosia (inability to recognize familiar stimuli), or disturbance in executive functioning (problems in planning, organizing, sequencing, abstracting). Dementia significantly interferes with work and social activities, and involves a decline from previous levels of functioning. Symptoms of dementia are generally permanent and progressive. In this example, the man's symptoms result from his post-operative condition and will most likely remit. They are thus more suggestive of delirium. Anterograde amnesia (Response 1) is characterized by the inability to recall information learned after a particular event or trauma. Retrograde amnesia (Response 2) is characterized by the inability to recall information learned before an event or trauma. While this man is exhibiting memory disturbance, he is also experiencing hallucinations, which are not involved in retrograde or anterograde amnesia.

1.) The multitrait-multimethod matrix is conducted in order to establish: a. content validity. b. discriminant validity. c. face validity. d. predictive validity.

a. discriminant validity. a. The multitrait-multimethod matrix is used to determine a test's construct validity, which requires both discriminant (divergent) validity, as well as convergent validity. Content validity (Response 1) is established by having a panel of experts look at the test items, and determine whether the test is adequately measuring the content domain it is purporting to measure. Face validity (Response 3) reflects the subjective experience of the test takers, as to whether the test appears to be measuring what is says it is. For example, the Rorschach has poor face validity, in that many people find it hard to believe that responses to inkblots can give detailed information about personality functioning. Predictive validity (Response 4) is a subtype of criterion-related validity, and it is established by correlating scores on the predictor test with some criterion outcome scores obtained later in time.

1.) A father brings in his daughter for psychological testing in order to assess her intellectual and academic potential. When discussing the results, you would be advised to do all of the following except: a. discuss any limitations of the tests. b. discuss the specific test scores obtained. c. discuss the results in terms of the child's strengths and weaknesses. d. discuss appropriate recommendations based on the results.

a. discuss the specific test scores obtained. a. When discussing the results of psychological tests, it is generally inadvisable to discuss specific test scores (e.g., IQ scores). It is more appropriate to talk about the confidence interval or the range that the child's IQ falls in, as well as to provide information about the significance of the scores. When discussing results, it is important to address any limitations of the test (Response 1), strengths and weaknesses of the child (Response 3), as well as recommendations that emerge from the results of the testing (Response 4).

1.) Five teams enter a ski jump competition. Each team is allowed up to four members. Each team member is allowed only one jump, and the team with the member who jumps the farthest is declared the winning team. This is an example of a(n): a. disjunctive task. b. conjunctive task. c. aggregate task. d. additive task.

a. disjunctive task. a. In a disjunctive task, the final product is the performance of the best member of the group. Here, the team's performance is judged by the performance of the best member of the group, or the best jump. The other jumps are not taken into consideration. In a conjunctive task (Response 2), by contrast, the group's accomplishment is limited by the least effective member's performance. This would be the case if the group's overall rating in the ski jump competition was that of its worst jumper. An additive task (Response 4) is one in which group members' separate performances are added to produce a combined effect. This would be the case if every member's jump was totaled to get an overall group score. An aggregate (Response 3) is not a task, but a group of people who are not aware of each other and do not interact with one another.

1.) Frequently patients report significant positive changes within the first few sessions of therapy. This improvement is likely: a. to manifest as a reduction in somatic complaints. b. due to an increased sense of hope. c. related to insights obtained in therapy. d. a placebo effect.

a. due to an increased sense of hope. a. Some meta-analytic studies (e.g., Howard et al) have found 50% of patients measurably improved within eight sessions, and 75% improved within six months. Somatic complaints (Response 1) are least likely to change dramatically, given that patients with psychosomatic disorder are among the most difficult to treat. Patients appear to improve in therapies that are not insight-oriented, such as behavioral therapy (Response 3). The patient improvements found appear to be genuine, due to increased hopefulness, rather than to a placebo effect.

1.) It has been argued that memory problems in the elderly are due more to difficulties with retrieval than encoding. Support for this contention is based on the fact that: a. elderly persons do better on tasks of recognition than recall. b. elderly persons exhibit declines in short-term memory. c. elderly persons perform better on ecological tasks of memory than tasks presented in memory labs. d. elderly persons perform better on memory tasks involving biographical information.

a. elderly persons do better on tasks of recognition than recall. a. It is hypothesized that memory problems in the elderly are due more to problems with retrieval than with encoding. In other words, the elderly can register information, but have difficulty with free recall, or bringing up the information from their memory banks. Giving them cues, such as on recognition tasks, can help them to remember the information. Short-term memory tends to remain intact in the elderly (Response 2). Response 3 is an accurate statement, however, it does not address the question. Response 4 is also a correct statement, yet it too does not answer the question. Instead, it states that the elderly can retrieve information from long-term memory, and it does not address the issue of retrieval versus encoding. Note that typically the elderly experience difficulty with retrieval and learning of new information.

1.) Dr. Z. is referred a patient by a managed care company for a brief evaluation. Due to contract constraints, the patient is referred out for treatment after the evaluation. The treating therapist meets with Dr. Z. and pays Dr. Z. a fee. In this case, the fee is: a. unethical, because fees should not be based on referrals. b. ethical, as long as this is a one-time arrangement. c. ethical, as long as the fee is based on the consultation. d. ethical, because the psychologist performed the evaluation.

a. ethical, as long as the fee is based on the consultation. a. According to the Ethics Code, any fee splitting between a psychologist and another professional should be based on services provided (e.g., consultative, administrative or other), and not the referral itself. Here, the fee would be ethical only if it were based on a consultation with Dr. Z.

1.) Graduate students Sally and Jennifer collaborate with a faculty member on a large research project. Both Sally and Jennifer write their master's theses based on parts of this research. When the faculty member submits an article based on this research, he lists himself as first author. This faculty member has acted a. unethically, because students should always receive first authorship on articles based on their master's theses. b. unethically, because two separate articles should have been authored. c. ethically, if the publication credits are an accurate representation of the contributions made by each of the researchers. d. ethically, if the faculty member discussed publication credits with Sally and Jennifer early in the process, and they were agreeable to this arrangement.

a. ethically, if the publication credits are an accurate representation of the contributions made by each of the researchers. a. Students generally should get first authorship on articles based on their dissertation, but the 2002 APA Ethics Code does not make this requirement for articles based on master's theses (ruling out Response 1). Often, a master's thesis is completed as a part of a larger on-going research project, and the student's contribution doesn't actually warrant first authorship. Similarly, the projects may not have been comprehensive enough to warrant separate articles being authored (Response 2). Although publication credits should be discussed as early as feasible, it is never enough for all parties to be agreeable to an arrangement (Response 4). The key provision is that publication credits must always "accurately reflect the relative scientific or professional contributions of the individuals involved, regardless of their relative status."

1.) A client executes an appropriate release of information, and requests that his test data be sent to an attorney. Dr. Assessor sends the attorney all of the test data, including scoring sheets, raw scores, and his own notes of in-session client behavior. Dr. Assessor has acted: a. unethically, because he included raw data, and raw data may never be released to an individual who lacks the competence to interpret them. b. ethically. c. unethically, because he should only have released a summary of the tests results to the attorney, and the raw data directly to the client. d. ethically, as long as the release form included information about the possible risks of releasing raw test data to unqualified individuals.

a. ethically. a. According to the 2002 APA Ethics Code, psychologists provide test data to the client or to other persons identified in the release signed by the client (ruling out Response 3). The Ethics Code's definition of test data includes raw and scaled scores, client responses to test questions, and in-session notes. In contrast, the 1992 Ethics Code only permitted the release of raw data to individuals who had the competence to understand and interpret them (Response 1), or to the client directly if the client insisted on receiving the data. There is no ethical requirement to inform clients of the risks of releasing raw data to individuals without the necessary competence (Response 4).

1.) A researcher who is studying attitudes toward abortion surveys a sample of African Americans, Hispanics, Asians, and Caucasians in four geographic regions (West, East, South and the Midwest). The researcher is doing so to optimize: a. external validity. b. internal validity. c. construct validity. d. reliability.

a. external validity. a. External validity refers to the generalizability of a study. Here, the researcher wants to sample several diverse groups from different regions so that the results of her study are not just applicable to her sample, but to the broader population. Internal validity (Response 2) has to do with whether the independent variable caused the changes in the dependent variable, or whether the changes were due to other factors such as history, regression, or selection. Construct validity (Response 3) refers to a test's adequacy in measuring the hypothetical trait or construct it is attempting to measure. Sampling diverse populations would not address issues of construct validity. Reliability (Response 4) has to do with repeatability or consistency of measurement.

1.) Habituation is distinguished from extinction in that: a. extinction involves repeated exposure to a US until it no longer elicits the UR. b. habituation involves repeated exposure to a CS until the CS no longer elicits the CR. c. extinction involves repeated exposure to a CS until it no longer elicits the CR. d. habituation occurs when a primary reinforcer loses its reinforcing quality from being presented too much.

a. extinction involves repeated exposure to a CS until it no longer elicits the CR. a. Extinction is brought about when a person is repeatedly exposed to the CS without the US being present. Habituation involves repeated exposure to a US until the US no longer elicits a UR (Response 1). For example, a person habituates to living in a noisy neighborhood, and no longer wakes up during the night when cars honk or loud music is played. Response 4 describes satiation, which results from excessive reinforcement, to the point that the reinforcer loses value (e.g., rewarding a child with candy all day long until the candy loses reinforcement value).

1.) In a study of the effects of psychiatric symptoms on work performance, it was found that hallucinations and insomnia were orthogonal. One can therefore conclude that: a. hallucinations and insomnia were correlated. b. hallucinations and insomnia were not correlated. c. both hallucinations and insomnia were predictive of work performance. d. hallucinations and insomnia should not both be entered into a regression equation because of potential problems with multicollinearity.

a. hallucinations and insomnia were not correlated. a. When variables are "orthogonal" they are not correlated with each other. When they are "oblique" they are correlated. These terms are more commonly used to describe the types of rotations available in factor analysis. An orthogonal rotation results in uncorrelated factors whereas an oblique rotation results in correlated factors. Multicollinearity (Response 4) occurs when predictors are highly correlated with each other, which is the opposite of what is occurring here.

1.) Adolescents go through stages in order to form a mature identity. An adolescent who is at the identity status of foreclosure would: a. be committed to a goal after having explored several alternatives. b. be in the process of exploration but not yet committed to a goal. c. have accepted a ready-made identity that an authority figure may have chosen. d. not be committed to clear goals, nor be actively trying to reach them.

a. have accepted a ready-made identity that an authority figure may have chosen. a. Researchers (e.g., Marcia) have grouped adolescents into four categories, which depict the progress made toward formulating a mature identity: identity achievement, moratorium, foreclosure, and diffusion. The person who is at the identity achievement status has explored several options before developing his or her own set of goals and values (Response 1). The person at the moratorium status (Response 2) is someone who has not made a definite commitment and is in the process of exploration. An adolescent who is at the identity status of foreclosure has committed to a goal without exploring other alternatives. This adolescent has accepted a ready-made identity chosen by an authority figure (e.g., an adolescent is planning on becoming a lawyer because her mother and father are both lawyers and they believe that she should also be a lawyer). The identity diffusion status describes someone who lacks direction, is not committed to goals and values, and is not trying to develop goals (Response 4).

1.) A child is generally unresponsive when called by name by teachers or peers in school, and tends to speak out of turn in a loud yet somewhat inarticulate fashion. This child should be evaluated for: a. hearing problems. b. Autism. c. ADHD. d. seizure disorder.

a. hearing problems. a. A child with such behavior, especially speaking in a loud yet inarticulate fashion, would need to be evaluated for a hearing impairment. An autistic child (Response 2) would not typically speak out of turn in a loud fashion. Furthermore, his or her speech would not be inarticulate, as it would be with a child who has a hearing impairment. While ADHD (Response 3) involves speaking out of turn and sometimes unresponsiveness to others, it does not involve impairments in speech and language. Seizure disorder (Response 4) is not typically characterized by these symptoms.

1.) A researcher believes that birth order significantly affects level of professional achievement; however, her study fails to find any significant results. She plans to revise her study. To increase her chances of finding significance, she should do all of the following except: a. increase sample size. b. use a one-tailed test. c. increase alpha. d. increase beta.

a. increase beta. a. The chances of finding significant results (also known as power) are enhanced when sample size is large (Response 1), a one-tailed test is used (Response 2), and alpha is high (Response 3). Beta error is defined as the failure to find group differences when group differences actually do exist. Thus, the higher beta is, the less likely one is to find significance. Also, since Power = 1 - Beta, increasing beta will have the effect of directly decreasing power.

1.) In spite of the fact that a father tells his young children that he loves them, he rarely smiles at them, and never hugs or kisses them. The children feel unloved based on their interpretation of the their father's: a. kinesics. b. proxemics. c. paralanguage. d. immersion-emersion.

a. kinesics. a. Kinesics describes body movements, such as facial expressions, gestures, physical contact, etc. Kinesics is different from proxemics (Response 2), which refer to the perception and use of personal space (e.g., how far one needs to stand from others when speaking to them). Paralanguage (Response 3) refers to vocal cues, such as loudness, pauses, rate of speech, silences, and inflections. Immersion-emersion (Response 4) refers to a stage in Cross's model of Black identity development.

1.) According to Rehm, depression is caused by: a. adopting maladaptive behaviors based on observing one's environment. b. adopting a negative view of one's self, the future, and the environment. c. negative evaluations of one's behavior, excessive punishment and little reinforcement applied by oneself. d. experiencing distressing events, negative interpretations of those events, and maladaptive responses.

a. negative evaluations of one's behavior, excessive punishment and little reinforcement applied by oneself. a. Rehm views depression and its concomitant low rate of behavior (e.g., lack of involvement in activities) as a result of negative self-evaluations, lack of self-reinforcement and high rates of self-punishment. Response 1 involves observation and modeling, and is more consistent with Social Learning Theory. Response 2 describes the negative cognitive triad proposed by Beck in his theory of Cognitive Therapy. Response 4 is consistent with rational emotive behavioral therapy (REBT) proposed by Ellis

1.) Edwin, a four-year-old boy, often throws a tantrum when he does not get his way. His parents consult with a psychologist who tells them to have Edwin sit on the stairs by himself for ten minutes when he begins to throw a tantrum. This psychologist is recommending that the parents use: a. extinction. b. positive punishment. c. negative reinforcement. d. negative punishment

a. negative punishment b. The psychologist is recommending that the parents put Edwin in time out, which involves the removal of all opportunities for reinforcement for a specific amount of time after the occurrence of an undesirable behavior, in order to reduce or eliminate that behavior. Time out is a type of negative punishment (Response 4). Extinction (Response 1) involves withholding reinforcement for a behavior that was previously reinforced. In extinction, reinforcement is continuously withheld in order to eliminate a behavior, whereas in time out, opportunities for reinforcement are removed for only a specified period of time. Positive punishment (Response 2) involves applying an aversive stimulus (e.g., yelling, slapping) after an undesirable behavior. Negative reinforcement (Response 3) involves removing an aversive stimulus after the desired behavior is emitted.

1.) According to the "Big 5" model, the five basic personality traits are: a. neuroticism, conscientiousness, agreeableness, extroversion, openness to experience. b. introversion/extroversion, stability/instability, normality/psychoticism, less intelligent/more intelligent, trusting/suspicious. c. reserved/outgoing, affected by feeling/emotionally stable, submissive/dominant, practical/imaginative, self-assured/apprehensive. d. social, conventional, artistic, enterprising, investigative.

a. neuroticism, conscientiousness, agreeableness, extroversion, openness to experience. a. The Five-Factor Theory has been developed by Costa and McCrae. Relying on the process of factor analysis, these researchers have concluded that the five basic factors in personality include openness to experience (O), conscientiousness (C), extroversion (E), agreeableness (A), and neuroticism (N). Note that these words form the acronym OCEAN. Response 2 describes Eysenck's three personality dimensions, which are introversion-extroversion, stability-instability, and normality-psychoticism, as well as two of Cattell's source traits. Response 3 describes five of Cattell's 16 source traits. Response 4 describes five of Holland's six occupational themes.

1.) According to behaviorists, depression is the result of: a. not enough reinforcement from the environment. b. too much punishment from the environment. c. maladaptive thinking. d. genetic predisposition.

a. not enough reinforcement from the environment. a. The traditional behaviorist view asserts that depression is the result of a low rate of response-contingent reinforcement, and that depression improves when the rate of reinforcement for adaptive behavior is increased. While it is true that too much punishment can contribute to depression (Response 2), this model states that it is more likely to be caused by inadequate reinforcement, which makes Response 1 a better answer. Cognitive-behaviorists believe depression results from maladaptive thinking (Response 3). While depression does have a significant genetic loading (concordance rates for identical twins range from 55 - 60%, and rates for fraternal twins and siblings are about 20%), this is not likely to be the attribution made by behaviorists (Response 4).

1.) Cri-du-chat is: a. Inherited and caused by a chromosomal addition b. Inherited and caused by a chromosomal deletion c. Not inherited and caused by a chromosomal addition d. not inherited and caused by a chromosomal deletion

a. not inherited and caused by a chromosomal deletion b. A difficult question! Cri-du-chat (cat's cry) syndrome is a non-inherited condition caused by a chromosomal deletion. Cri-du-chat is characterized by mental retardation and delayed development, distinctive facial features, small head size, low birth weight, and weak muscle tone in infancy. Some children can also have a heart defect. The disorder is named because of the high-pitched cry, resembling that of a cat, which infants with the condition often exhibit. The most well-known disorder caused by chromosomal addition is Down syndrome (Trisomy 21) caused by an extra chromosome 21 from either the egg or sperm cell. Down syndrome is also not inherited; as a rule, most disorders caused by chromosomal abnormalities are not inherited

1.) As opposed to normative conformity, informational conformity is most likely to: a. occur when the task is ambiguous. b. occur when minority members perceive themselves as similar to the majority. c. result in public compliance but not private acceptance. d. take place when the group does not have consensus.

a. occur when the task is ambiguous. a. A normative influence on conformity occurs when people comply with the majority's standards for acceptable behavior because they want to be liked. By contrast, informational conformity refers to the tendency for people to conform to the majority decision because they want to perceive reality correctly. Both normative and informational conformity are more likely to occur when the majority group has consensus, ruling out Response 4. Normative conformity occurs when members perceive themselves as similar to the majority (Response 2), when they are in cohesive groups, or when there is a fear of rejection. Normative conformity results in public compliance, but not in private acceptance (Response 3). For example, in the Asch line-judging study, 76% of the participants gave at least one "obviously incorrect answer" when asked to give their responses aloud (public compliance), while subjects given the opportunity to write their answers down (private acceptance) did not give incorrect answers. Informational conformity is most likely to occur when the task is difficult and ambiguous (Response 1). It is least likely to occur when the task is relatively simple because then the person feels confident that reality is being perceived accurately.

1.) The biserial coefficient is appropriate for examining the relationship between: a. one continuous variable and one dichotomous variable. b. two dichotomous variables. c. reliability and validity. d. three or more variables.

a. one continuous variable and one dichotomous variable. a. The biserial coefficient is a type of correlation coefficient used when one variable is continuous (i.e., interval or ratio data) and one variable is dichotomous (i.e., nominal data). More specifically, a "point biserial coefficient" is used when the dichotomous variable is a true dichotomy (i.e., naturally occurring such as male/female) while a "biserial coefficient" is used when the dichotomous variable is an artificial dichotomy (e.g., high income/low income). A phi or tetrachoric coefficient is appropriate when both variables are dichotomous (Response 2). The phi is used for true dichotomies and the tetrachoric for artificially occurring ones. A multiple correlation (Multiple R) examines the relationship between two or more predictor variables and one continuous criterion variable, and a canonical correlation examines the relationship between two or more predictor variables and two or more criterion variables (Response 4). There is no coefficient that describes the degree of correlation between reliability and validity (Response 3).

1.) Elements of overcorrection include: a. physical guidance and negative reinforcement. b. physical guidance and reparation. c. reparation and punishment. d. restitution and positive reinforcement.

a. physical guidance and reparation. a. Overcorrection is classified as a method of punishment involving restitution or reparation, as well as physical guidance. It is most commonly used to decrease inappropriate behaviors. Restitution refers to having the person make up for an inappropriate behavior, e.g., if an adolescent makes a mess in her room, she is required to clean her room and maybe even other rooms in the house. Physical guidance may be necessary to help the person complete the different steps of the task. Overcorrection does not involve negative reinforcement (Response 1), which is the removal of an aversive stimulus to increase a behavior. While overcorrection is a type of punishment, Response 3 is too general. Positive reinforcement (Response 4), which is the application of a positive stimulus to increase a behavior, is not involved in overcorrection

1.) A patient whose corpus callosum has been severed is presented with an object in her left visual field. Most likely she will be able to: a. pick out the object with her right hand. b. identify the object by name. c. name the object and pick it out with her left hand. d. pick out the object with her left hand.

a. pick out the object with her left hand. b. When the corpus callosum is severed, there is no communication between the left and right hemispheres, each of which seems to have specialized functions. Also, the cortex has contralateral representation, meaning that the right hemisphere controls the left side of the body, and vice versa. When information is presented in the left visual field, it is processed in the right hemisphere. The right hemisphere controls the left side of the body, therefore the person can pick out the object with her left hand only, and not the right hand (Response 1). The person cannot name the object since language abilities are housed in the left hemisphere, and not the right (Responses 2 and 4).

1.) A person who has been diagnosed with deep dyslexia would be most likely to: a. read "tap" instead of "pat." b. read "come" as "comb." c. read "couch" instead of "chair." d. read "strong" as "stroke."

a. read "couch" instead of "chair." a. Deep dyslexia typically involves semantic errors; in other words, the person with deep dyslexia mistakenly reads a given word as one with similar meaning. Response 2 describes surface dyslexia. In surface dyslexia, the person cannot recognize words, and instead, sounds them out. Therefore, irregular words, such as "come," are mispronounced. ("Come" is an irregular word because it is pronounced differently than other words which are spelled in a similar fashion, such as "dome" or "home"). When suffering from neglect (Response 4) the person misreads the first or last half of the word.

1.) A patient who experiences mild head trauma is most likely to show deficits in: a. sensory memory. b. procedural memory. c. recent memory. d. remote memory.

a. recent memory. a. Retrograde amnesia is common following mild head trauma. This amnesia is usually for events just prior to the trauma, or recent memory. Some theorists divide long-term memory into recent memory, which lasts around two weeks, and remote memory (Response 4), which lasts two years or more. Sensory memory (Response 1) involves transforming sensory input into the type of data that can be understood, and storing the information as a visual or auditory image. Procedural memory (Response 2) involves the recollection of skills, physical operations, and procedures that can be remembered automatically without conscious awareness (e.g., riding a bike). Procedural memories are not stored with respect to specific times or places.

1.) You walk into a restaurant and smell a meal that reminds you of a gathering in your grandmother's house when you were five. This is an example of: a. redintegration. b. a flashbulb memory. c. retrieval. recognition

a. redintegration. a. This question is describing the process of redintegration, which refers to a type of remembering that occurs when something (e.g., a smell) unlocks a rapid chain of memories. Flashbulb memories (Response 2) involve memories of distinct, significant events, usually of a traumatic nature, which is not the case in this question. Retrieval (Response 3) refers to the process of accessing information from long-term memory. While the situation described here does involve accessing information, retrieval is too general of a response. Recognition (Response 4) describes the process of matching a specific cue to items in long-term memory; again, while some recognition may be involved here, it is also too general of a response.

1.) When using a predictor test for hiring decisions, a psychologist involved in the hiring process for a company would be most likely to increase the predictor cutoff in order to: a. increase the number of successful employees who are hired based on the test. b. reduce the number of unsuccessful employees who are hired based on the test. c. increase the number of unsuccessful employees who are not hired based on the test. d. decrease the number of successful employees who are not hired based on the test.

a. reduce the number of unsuccessful employees who are hired based on the test. a. Typically one would choose to raise the predictor cutoff to reduce the number of false positives, those employees who do well on the test but actually perform poorly in the company. Raising the predictor cutoff would also result in reducing the true positives, those employees who do well on the test and perform well at work (Response 1). Increasing the predictor cutoff also has the effect of increasing both the true negatives (Response 3) and the false negatives (Response 4). While Response 3 is a true statement, it typically is not the reason one would choose to raise the predictor cutoff.

1.) In terms of attachment, early deprivation has been found to have: a. mostly irreversible consequences, and to have progressive impact on the child's interpersonal relationships. b. mostly irreversible consequences, however, the effects may be somewhat mitigated depending on the amount of nurturance received later. c. reversible consequences, depending on the child's cognitive ability. d. reversible consequences, depending on the nature of the child's subsequent relationships.

a. reversible consequences, depending on the nature of the child's subsequent relationships. b. Even though Bowlby suggested that disruption of the attachment bond has irreversible effects, there is evidence that the impact of early deprivation on the attachment bond can actually be reversible. The quality of the care that the child later receives is the most significant factor in whether the child will be able to recover from early deprivation.

1.) A person is able to watch the news on television while the radio is playing and a conversation is being held in the background. This is an example of: a. dichotic listening. b. sustained attention. c. selective attention. d. simultaneous attention.

a. selective attention. a. Selective attention refers to the ability to select, and focus on, one stimulus. Here, the person is watching the news, while tuning out the noise in the background. Dichotic listening tasks (Response 1) involve listening to different sounds presented to each ear simultaneously. Sustained attention (Response 2) involves the ability to focus and maintain attention on a task. Simultaneous attention (Response 4) describes paying attention to different stimuli at the same time.

1.) Dissociative identity disorder can most likely be attributed to: a. severe, chronic childhood abuse. b. a traumatic event. c. poor integration of split-off parts of the self. d. authoritarian parenting.

a. severe, chronic childhood abuse. a. Severe, chronic abuse during childhood is thought to contribute to the development of dissociative identity disorder (DID). DID involves two or more distinct identities or personality states with a failure to integrate aspects of identity, memory, and consciousness. A single traumatic event (Response 2) would be unlikely to lead to dissociative identity disorder, however, it could result in acute stress disorder or PTSD. Poor integration of split-off parts of the self (Response 3) is more descriptive of borderline personality disorder. Authoritarian parenting (Response 4) is too general, and would not in and of itself contribute to DID.

1.) Most research supports the idea that children who watch more television are more aggressive. This finding is best explained by: a. social learning theory. b. frustration-aggression hypothesis. c. social influence. d. deindividuation.

a. social learning theory. a. Social Learning Theory asserts that people learn to be aggressive through modeling, or observing others engage in aggressive behavior. The frustration-aggression hypothesis (Response 2) states that a specific frustrating event leads to aggression. Social influence (Response 3) refers specifically to the effect of others on a person in terms of conformity and obedience. Deindividuation (Response 4) can lead to increased aggression because of anonymity involved in being a part of a group, but not because of watching others.

1.) In a research laboratory, a ten-month-old crawls across the visual cliff only when the mother is present at the other end, with her arms open and smiling at the baby. This is an example of: a. the development of depth perception. b. social referencing. c. modeling. d. secure attachment.

a. social referencing. a. Social referencing refers to a person's use of cues from another person to deal with affective uncertainty (e.g., whether a situation is safe or dangerous). This begins to occur before one year of age. Another example of social referencing is when a child falls, and looks at the parent before deciding whether or not to cry. If the parent looks alarmed, the child cries. If the parent seems calm, frequently the child does not cry. Visual cliff studies have been used extensively to look at the development of depth perception (Response 1), but this question is not asking about depth perception. The mother is not modeling the behavior of crossing the visual cliff herself (Response 3). Secure attachment (Response 4) refers to a warm bond between infant and caretaker, and is not the focus of this question

1.) A ten-year-old repeatedly awakens from sleep with a loud scream, and shows signs of fear and autonomic arousal. She is confused and disoriented, and unresponsive to others. Later, however, she does not remember the episode. These episodes are most likely occurring during _______ sleep, and are consistent with a diagnosis of _________. a. REM, sleep terror disorder. b. REM, nightmare disorder. c. stage 4, sleep terror disorder. d. stage 4, nightmare disorder.

a. stage 4, sleep terror disorder. a. The symptoms listed describe sleep terror disorder. Sleep terror disorder, a parasomnia, occurs during stage 4 sleep, typically in the first third of the night when such deep sleep predominates. Nightmare disorder (Responses 2 and 4) involves repeated awakening from sleep with detailed recall of frightening dreams. On awakening, the person is alert and oriented, in contrast to the confusion and disorientation seen in sleep terror disorder. Nightmare disorder occurs during REM sleep, most often later in the night, which is when REM sleep predominates.

1.) Psychologists' professional behavior is regulated by: a. the courts. b. state and local psychological associations. c. state boards. d. the Ethics Code.

a. state boards. a. The professional behavior of psychologists is regulated by state boards (e.g., in California, the Board of Psychology). State boards grant licenses to practice, and have the authority and responsibility to investigate and respond to allegations of inappropriate (illegal and/or unethical) professional conduct. The courts (Response 1) do not directly regulate psychologists' conduct, though they become involved in cases of malpractice, lawsuits, etc. State and local psychological associations (e.g., the California Psychological Association or the Los Angeles County Psychological Association) are not regulatory bodies (Response 2). Even though they have codes of conduct, they are professional associations or groups. The Ethics Code (Response 4) sets guidelines for ethical behavior of psychologists, but in and of itself doesn't regulate behavior.

1.) A child has a history of engaging in driven, repetitive, nonfunctional behaviors, such as body rocking and self-hitting. These symptoms are most characteristic of: a. autistic disorder. b. chronic motor tic disorder. c. stereotypic movement disorder. d. obsessive-compulsive disorder.

a. stereotypic movement disorder. a. Stereotypic movement disorder is characterized by motor behavior that is repetitive and nonfunctional. This diagnosis is made only if the repetitive behaviors cannot be accounted for by another diagnosis, such as OCD or a tic disorder. While repetitive movements are present in autistic disorder (Response 1), other symptoms must be present in order for the diagnosis to be made, such as impairment in social interactions and communication, and a restricted repertoire of behavior. In stereotypic movement disorder, the movements tend to be driven or intentional, whereas in tic disorders (Response 2), the tics have a more involuntary quality. Common examples of motor tics include eye blinking, touching, squatting and retracing steps. In obsessive-compulsive disorder (Response 4), the compulsions are more complex and ritualistic, and are performed in response to obsessions or rules, for the purpose of reducing distress. By contrast, in stereotypic movement disorder, the behaviors are non-functional.

1.) A psychologist who has been employed in the prison system in California has a traumatic experience with her immediate supervisor who continually harasses and criticizes her. She quits the job, moves out of state, and is offered a job in a prison in Hawaii, which she declines, feeling it will be too traumatic. The factor most likely to be operating here is: a. stimulus generalization. b. the law of effect. c. superstitious behavior. d. covert sensitization.

a. stimulus generalization. a. This scenario describes stimulus generalization, or the tendency to generalize from a conditioned stimulus (the prison in California) that had been paired with harassing behavior by her boss, to a neutral yet similar stimulus (a prison in Hawaii). Thorndike's revised law of effect (Response 2) relates to the increased emitting of behaviors that have been reinforced. The earliest version stated that behaviors that are punished are less likely to be emitted; even though the psychologist may have been punished by her boss, there is no mention of her behavior changing. Superstitious behavior (Response 3) results from noncontingent reinforcement of a behavior, and has the effect of increasing a behavior. Covert sensitization (Response 4) is another name for aversive counterconditioning, and involves pairing an aversive CS with a pleasurable CS in imagination (e.g., imagining oneself smoking and developing lung cancer).

1.) After cardiac bypass surgery, a 60-year-old man experiences a sudden disturbance of consciousness, with reduced ability to focus, sustain, or shift attention. Appropriate interventions would include any of the following except: a. use of an anti-psychotic medication. b. placing a television set in the patient's room. c. temporarily prohibiting family members from visiting. d. supportive therapy including fluid and electrolyte maintenance.

a. temporarily prohibiting family members from visiting. a. The symptoms described here suggest delirium. In treating delirium, it is important to attempt to reorient the patient. Contact with close family members could facilitate reorientation. The television (Response 2) can also be a useful tool for reorientation to time and day. The drug of choice for delirium (the agitated delirious patient) is Haldol, which is an antipsychotic (Response 1). Fluid and electrolyte maintenance (Response 4) would also be important components of treatment.

1.) A significant concern in prescribing methylphenidate to children is: a. the tolerance to the medication that develops may require increasing dose level over time. b. sustained growth suppression. c. temporary growth suppression, which can be alleviated with drug holidays. d. temporary growth suppression, which can be alleviated by reducing drug dose.

a. temporary growth suppression, which can be alleviated with drug holidays. a. Methylphenidate (Ritalin), frequently prescribed for children with ADHD, may decrease appetite and temporarily suppress growth. For this reason it is frequently suggested that children taking stimulants receive drug holidays (periods such as weekends and summer vacations when they are not given the medication). Drug holidays allow children the opportunity to compensate for any growth suppression.

1.) According to Piaget, constructivism refers to: a. the child's ability to recognize that an object can retain its basic characteristics even though it might change form. b. the child's ability to gain new knowledge by interacting with others. c. the child's ability to gain new knowledge by interacting with objects and events in the environment. d. the child's ability to develop abstract thought and reflect about oneself.

a. the child's ability to gain new knowledge by interacting with objects and events in the environment. a. According to constructivism, a person develops new knowledge based on a foundation of previous learning, and by interacting with objects and events in the environment. When applied to schooling, constructivists place emphasis on the student, rather than the teacher. The teacher is a facilitator who assists students to construct their own conceptualizations and solutions to problems. Response 1 describes the ability to conserve. Response 2 is the opposite of constructivism, since it states that knowledge is transmitted by others, and not developed by the self. Response 4 describes the formal operational stage of Piaget's cognitive development, as well as metacognition.

1.) One aspect of scoring a Rorschach protocol includes developmental quality, which involves looking at: a. the degree of integration of the response. b. the degree of accuracy with which the shape of the response matches the blot. c. the type and quantity of specific subject categories perceived in the responses. d. the area of the blot that the person used to develop the response.

a. the degree of integration of the response. a. Developmental quality is determined by evaluating each response in terms of its degree of integration (+, v/+, o, v). Response 4 describes the location of the response (W, D, Dd, S). Form quality (Response 2) refers to how accurately the percept relates to the form of the inkblot (+, o, u, -). Content (Response 3) includes the specific categories the responses belong to (e.g., Clothing, Science, Whole Human, etc.).

1.) What does the Ethics Code say with regard to sexual relationships with former therapy clients, and therapy with former sexual partners? a. Both may be ethically permissible depending on the circumstances. b. Both are considered unethical conduct. c. the former may be permissible, while the latter is absolutely prohibited. d. The latter may be permissible, but the former is absolutely prohibited.

a. the former may be permissible, while the latter is absolutely prohibited. a. While the prohibition against therapy with former sexual partners is absolute, the prohibition against sexual relationships with former therapy clients is not as clear cut. With regard to sexual relationships with former therapy clients, the Ethics Code states that psychologists may not have sex with a former patient unless at least two years have passed since treatment ended and the "most unusual circumstances" exist. The burden is on the psychologist to prove that there has been no exploitation, especially in light of several factors. Therefore, it is possible for such a relationship to be ethically permissible. By contrast, the provision regarding therapy with previous sexual partners is absolute: psychologists may never treat previous sexual partners.

1.) You read the section of your materials that describes the stages of memory. Then you take this practice test and remember all of the material. However, when you go to take the licensing exam four weeks later and a question comes up in this area, you "draw a blank." The most likely explanation is that: a. this is a manifestation of the Zeigarnik effect. b. the information did not reach long-term memory because short-term memory capacity was exceeded. c. the information reached long-term memory, however, difficulty with memory accessing was occurring. d. the information reached long-term memory, however, problems with proactive interference were being manifest.

a. the information reached long-term memory, however, difficulty with memory accessing was occurring. a. The fact that you were able to remember the information while taking the practice test indicates that the information reached long-term memory. Difficulty with recall on the exam suggests problems with retrieval, or memory accessing. It is possible that retroactive interference is occurring if information studied subsequent to the memory section (recently studied information) is interfering with the material studied previously. This scenario does not fit the description of proactive interference (Response 4), in that the previously learned information about memory is not interfering with more recently learned information. A manifestation of the Zeigarnik effect (Response 1) would be occurring if, for example, two days after taking the exam, the correct answer came to mind.

1.) Researchers such as Garmezy and Rutter have studied risk and protective factors in development. In designing interventions to reduce the impact of risk factors on development, one should keep in mind that: a. males and females demonstrate approximately equivalent degrees of resiliency throughout childhood and adolescence. b. there is a positive correlation between severity of risk factors and developmental psychopathology. c. the interaction between risk and protective factors loses significance with increasing age. d. there is a positive correlation between number of risk factors and developmental psychopathology.

a. there is a positive correlation between number of risk factors and developmental psychopathology. b. There is a positive correlation between number of risk factors and developmental psychopathology. The overall risk of psychopathology can be determined by adding up the specific risk factors present, and calculating an adversity index. This procedure is based on the theory that most people can cope with a small number of risk factors, however, as risk factors increase, individuals begin to show signs of disturbance. The number of risk factors has been more consistently found to correlate with psychopathology than the severity of risk factors (Response 2). Males and females differ in terms of their resilience to risk factors (Response 1): boys appear more vulnerable to risk factors from the prenatal period to about age 10, while girls grow more vulnerable to risk factors during their teens. Also, girls generally appear to possess a wider range of coping skills than boys. Protective and risk factors continue to remain significant with increasing age (Response 3).

1.) A psychologist accepts into treatment the wife of her own therapist. According to APA's Ethical Principles of Psychologists and Code of Conduct (2002), the psychologist's behavior is: a. possibly ethical, because multiple relationships are not inherently unethical. b. unethical, because of the high likelihood that the psychologist's objectivity and competence will be impaired. c. unethical, because multiple relationships are unethical except "in the most unusual circumstances." d. possibly ethical, unless there is a risk of exploitation or harm to the client.

a. unethical, because of the high likelihood that the psychologist's objectivity and competence will be impaired. a. Although it is true that multiple relationships are not inherently unethical (Response 1), they are only acceptable if they do not impair the therapist's objectivity or risk harm to the patient. In this case, it is difficult to argue that a therapist could be objective when listening to the wife of her own therapist; thus there would be significant risk of harm (Response 4). The phrase "in the most unusual circumstances" (Response 3) does not apply to the issue of multiple relationships; it applies to sex with former clients.

1.) In comparing life expectancy between men and women, and between white and non-white Americans: a. men tend to outlive women, and whites tend to outlive non-whites. b. men tend to outlive women, and non-whites tend to outlive whites. c. women tend to outlive men, and non-whites tend to outlive whites. d. women tend to outlive men, and whites tend to outlive non-whites.

a. women tend to outlive men, and whites tend to outlive non-whites. b. In terms of life expectancy, women on average outlive men, and American whites on average outlive American non-whites.

1.) The color red is paired with a loud noise over several trials. The color red is then paired with a tone. Afterwards, the color red is again paired with the loud noise. This is an example of: a. stimulus generalization. b. pseudoconditioning. c. higher-order conditioning. d. simultaneous conditioning.

higher-order conditioning. a. This situation describes higher-order conditioning, in which a neutral stimulus (the color red) is deliberately paired with a US (the loud noise). Once the neutral stimulus begins to elicit a response, it is said to become a CS, and it is then paired with a second neutral stimulus (the tone). In order to keep the higher-order conditioning in effect, the first neutral stimulus (the color red) will usually be again paired with the US at some point. Stimulus generalization (Response 1) is a process that does not involve pairing, in which a person generalizes from a CS (e.g., the color red) to a similar neutral stimulus (e.g., the color pink). Pseudoconditioning (Response 2) occurs when a response is elicited to a neutral stimulus that has not been paired with either the CS or the US. Simultaneous conditioning (Response 4) involves presenting the CS and the US at the same time, such that they completely overlap, which results in no conditioning taking place.


Related study sets

Combo with "MGMT 468 Ch. 5 Quiz" and 1 other

View Set

Physics 211 Multiple Choice Final Prep

View Set

Exercise Biology Chapter 8 Review

View Set

ATTR 222 Ch 22 The Shoulder Complex: Recognition and Management of Specific Injuries

View Set

Chapter 4 - An Introduction to Stakeholder Analysis

View Set

Chapter 1: Using Technology to Change the World

View Set